127 Fundamentals of Mathematics: Part 9

0.219

Exercise 6.3.14

0.811

Exercise 6.3.15

(Solution on p. 405.)

4.8

Exercise 6.3.16

2.6

Exercise 6.3.17

(Solution on p. 405.)

16.12

Exercise 6.3.18

25.88

Available for free at Connexions <http://cnx.org/content/col10615/1.4>

349

Exercise 6.3.19

(Solution on p. 405.)

6.0005

Exercise 6.3.20

1.355

Exercise 6.3.21

(Solution on p. 405.)

16.125

Exercise 6.3.22

0.375

Exercise 6.3.23

(Solution on p. 405.)

3.04

Exercise 6.3.24

21.1875

Exercise 6.3.25

(Solution on p. 405.)

8.225

Exercise 6.3.26

1.0055

Exercise 6.3.27

(Solution on p. 405.)

9.99995

Exercise 6.3.28

22.110

For the following 10 problems, convert each complex decimal to a fraction.

Exercise 6.3.29

(Solution on p. 405.)

0.7 12

Exercise 6.3.30

0.012 12

Exercise 6.3.31

(Solution on p. 405.)

2.16 14

Exercise 6.3.32

5.18 23

Exercise 6.3.33

(Solution on p. 405.)

14.11213

Exercise 6.3.34

80.001137

Exercise 6.3.35

(Solution on p. 405.)

1.40 5

16

Exercise 6.3.36

0.8 53

Exercise 6.3.37

(Solution on p. 405.)

1.9 75

Exercise 6.3.38

1.7 37

9

Available for free at Connexions <http://cnx.org/content/col10615/1.4>

Image 291

350

CHAPTER 6. DECIMALS

6.3.4.1 Exercises for Review

Exercise 6.3.39

(Solution on p. 405.)

(Section 3.6) Find the greatest common factor of 70, 182, and 154.

Exercise 6.3.40

(Section 3.6) Find the greatest common multiple of 14, 26, and 60.

Exercise 6.3.41

(Solution on p. 405.)

(Section 4.5) Find the value of 3 · 15

.

5

18 ÷ 59

Exercise 6.3.42

(Section 5.4) Find the value of 52 + 8 1

3

12 .

Exercise 6.3.43

(Solution on p. 405.)

(Section 6.2) In the decimal number 26.10742, the digit 7 is in what position?

6.4 Rounding Decimals4

6.4.1 Section Overview

• Rounding Decimal Numbers

6.4.2 Rounding Decimal Numbers

We rst considered the concept of rounding numbers in Section 1.4 where our concern with rounding was related to whole numbers only. With a few minor changes, we can apply the same rules of rounding to decimals.

To round a decimal to a particular position:

1. Mark the position of the round-o digit (with an arrow or check).

2. Note whether the digit to the immediate right of the marked digit is (a) less than 5. If so, leave the round-o digit unchanged.

(b) 5 or greater. If so, add 1 to the round-o digit.

3. If the round-o digit is

(a) to the right of the decimal point, eliminate all the digits to its right.

(b) to the left of the decimal point, replace all the digits between it and the decimal point with zeros and eliminate the decimal point and all the decimal digits.

6.4.2.1 Sample Set A

Round each decimal to the specied position. (The numbers in parentheses indicate which step is being used.)

Example 6.15

Round 32.116 to the nearest hundredth.

(1)

4This content is available online at <http://cnx.org/content/m34959/1.2/>.

Available for free at Connexions <http://cnx.org/content/col10615/1.4>

Image 292

351

(2b) The digit immediately to the right is 6, and 6 > 5, so we add 1 to the round-o digit: 1 + 1 = 2

(3a) The round-o digit is to the right of the decimal point, so we eliminate all digits to its right.

32.12

The number 32.116 rounded to the nearest hundredth is 32.12.

Example 6.16

Round 633.14216 to the nearest hundred.

(1)

(2a) The digit immediately to the right is 3, and 3 < 5 so we leave the round-o digit unchanged.

(3b) The round-o digit is to the left of 0, so we replace all the digits between it and the decimal point with zeros and eliminate the decimal point and all the decimal digits. 600

The number 633.14216 rounded to the nearest hundred is 600.

Example 6.17

1,729.63 rounded to the nearest ten is 1,730.

Example 6.18

1.0144 rounded to the nearest tenth is 1.0.

Example 6.19

60.98 rounded to the nearest one is 61.

Sometimes we hear a phrase such as “round to three decimal places.” This phrase means that the round-o digit is the third decimal digit (the digit in the thousandths position).

Example 6.20

67.129 rounded to the second decimal place is 67.13.

Example 6.21

67.129558 rounded to 3 decimal places is 67.130.

6.4.2.2 Practice Set A

Round each decimal to the specied position.

Exercise 6.4.1

(Solution on p. 406.)

4.816 to the nearest hundredth.

Exercise 6.4.2

(Solution on p. 406.)

0.35928 to the nearest ten thousandths.

Exercise 6.4.3

(Solution on p. 406.)

82.1 to the nearest one.

Exercise 6.4.4

(Solution on p. 406.)

753.98 to the nearest hundred.

Exercise 6.4.5

(Solution on p. 406.)

Round 43.99446 to three decimal places.

Exercise 6.4.6

(Solution on p. 406.)

Round 105.019997 to four decimal places.

Exercise 6.4.7

(Solution on p. 406.)

Round 99.9999 to two decimal places.

Available for free at Connexions <http://cnx.org/content/col10615/1.4>

352

CHAPTER 6. DECIMALS

6.4.3 Exercises

For the rst 10 problems, complete the chart by rounding each decimal to the indicated positions.

Exercise 6.4.8

(Solution on p. 406.)

20.01071

Tenth Hundredth Thousandth Ten Thousandth

Table 6.1

Exercise 6.4.9

3.52612

Tenth Hundredth Thousandth Ten Thousandth

3.53

Table 6.2

Exercise 6.4.10

(Solution on p. 406.)

531.21878

Tenth Hundredth Thousandth Ten Thousandth

Table 6.3

Exercise 6.4.11

36.109053

Tenth Hundredth Thousandth Ten Thousandth

36.1

Table 6.4

Exercise 6.4.12

(Solution on p. 406.)

1.999994

Tenth Hundredth Thousandth Ten Thousandth

Table 6.5

Exercise 6.4.13

7.4141998

Available for free at Connexions <http://cnx.org/content/col10615/1.4>

353

Tenth Hundredth Thousandth Ten Thousandth

7.414

Table 6.6

Exercise 6.4.14

(Solution on p. 406.)

0.000007

Tenth Hundredth Thousandth Ten Thousandth

Table 6.7

Exercise 6.4.15

0.00008

Tenth Hundredth Thousandth Ten Thousandth

0.0001

Table 6.8

Exercise 6.4.16

(Solution on p. 406.)

9.19191919

Tenth Hundredth Thousandth Ten Thousandth

Table 6.9

Exercise 6.4.17

0.0876543

Tenth Hundredth Thousandth Ten Thousandth

Table 6.10

Calculator Problems

For the following 5 problems, round 18.4168095 to the indicated place.

Exercise 6.4.18

(Solution on p. 407.)

3 decimal places.

Exercise 6.4.19

1 decimal place.

Exercise 6.4.20

(Solution on p. 407.)

5 decimal places.

Available for free at Connexions <http://cnx.org/content/col10615/1.4>

354

CHAPTER 6. DECIMALS

Exercise 6.4.21

6 decimal places.

Exercise 6.4.22

(Solution on p. 407.)

2 decimal places.

Calculator Problems

For the following problems, perform each division using a calculator.

Exercise 6.4.23

4 ÷ 3 and round to 2 decimal places.

Exercise 6.4.24

(Solution on p. 407.)

1 ÷ 8 and round to 1 decimal place.

Exercise 6.4.25

1 ÷ 27 and round to 6 decimal places.

Exercise 6.4.26

(Solution on p. 407.)

51 ÷ 61 and round to 5 decimal places.

Exercise 6.4.27

3 ÷ 16 and round to 3 decimal places.

Exercise 6.4.28

(Solution on p. 407.)

16 ÷ 3 and round to 3 decimal places.

Exercise 6.4.29

26 ÷ 7 and round to 5 decimal places.

6.4.3.1 Exercises for Review

Exercise 6.4.30

(Solution on p. 407.)

(Section 1.2) What is the value of 2 in the number 421,916,017?

Exercise 6.4.31

(Section 2.4) Perform the division: 378 ÷ 29.

Exercise 6.4.32

(Solution on p. 407.)

(Section 3.2) Find the value of 44.

Exercise 6.4.33

(Section 4.3) Convert 11 to a mixed number.

3

Exercise 6.4.34

(Solution on p. 407.)

(Section 6.3) Convert 3.16 to a mixed number fraction.

6.5 Addition and Subtraction of Decimals5

6.5.1 Section Overview

• The Logic Behind the Method

• The Method of Adding and Subtracting Decimals

• Calculators

5This content is available online at <http://cnx.org/content/m34960/1.5/>.

Available for free at Connexions <http://cnx.org/content/col10615/1.4>

355

6.5.2 The Logic Behind the Method

Consider the sum of 4.37 and 3.22. Changing each decimal to a fraction, we have 4 37

100 + 3 22

100 Performing the addition, we get

4.37 + 3.22 = 4 37

100 + 3 22

100

=

4·100+37

100

+ 3·100+22

100

=

437

100 + 322

100

=

437+322

100

=

759

100

=

7 59

100

=

seven and fty-nine hundredths

=

7.59

Thus, 4.37 + 3.22 = 7.59.

6.5.3 The Method of Adding and Subtracting Decimals

When writing the previous addition, we could have written the numbers in columns.

4.37

+3.22

7.59

This agrees with our previous result. From this observation, we can suggest a method for adding and subtracting decimal numbers.

Method of Adding and Subtracting Decimals

To add or subtract decimals:

1. Align the numbers vertically so that the decimal points line up under each other and the corresponding decimal positions are in the same column.

2. Add or subtract the numbers as if they were whole numbers.

3. Place a decimal point in the resulting sum or dierence directly under the other decimal points.

6.5.3.1 Sample Set A

Find the following sums and dierences.

Example 6.22

9.813 + 2.140

9.813

The decimal points are aligned in the same column.

+2.140

11.953

Example 6.23

841.0056 + 47.016 + 19.058

Available for free at Connexions <http://cnx.org/content/col10615/1.4>

Image 293

Image 294

Image 295

Image 296

356

CHAPTER 6. DECIMALS

841.0056

47.016

+19.058

To insure that the columns align properly, we can write a 0 in the position at the end of the numbers 47.016 and 19.058 without changing their values.

Example 6.24

1.314 − 0.58

1.314

−0.58

Write a 0 in the thousandths position.

Example 6.25

16.01 − 7.053

16.01

−7.053

Write a 0 in the thousandths position.

Example 6.26

Find the sum of 6.88106 and 3.5219 and round it to three decimal places.

6.88106

+3.5219

Write a 0 in the ten thousandths position.

We need to round the sum to the thousandths position. Since the digit in the position immediately to the right is 9, and 9>5, we get

10.403

Example 6.27

Wendy has $643.12 in her checking account. She writes a check for $16.92. How much is her new account balance?

Available for free at Connexions <http://cnx.org/content/col10615/1.4>

Image 297

357

To nd the new account balance, we need to nd the dierence between 643.12 and 16.92. We will subtract 16.92 from 643.12.

After writing a check for $16.92, Wendy now has a balance of $626.20 in her checking account.

6.5.3.2 Practice Set A

Find the following sums and dierences.

Exercise 6.5.1

(Solution on p. 407.)

3.187 + 2.992

Exercise 6.5.2

(Solution on p. 407.)

14.987 − 5.341

Exercise 6.5.3

(Solution on p. 407.)

0.5261 + 1.0783

Exercise 6.5.4

(Solution on p. 407.)

1.06 − 1.0535

Exercise 6.5.5

(Solution on p. 407.)

16,521.07 + 9,256.15

Exercise 6.5.6

(Solution on p. 407.)

Find the sum of 11.6128 and 14.07353, and round it to two decimal places.

6.5.4 Calculators

The calculator can be useful for nding sums and dierences of decimal numbers. However, calculators with an eight-digit display cannot be used when working with decimal numbers that contain more than eight digits, or when the sum results in more than eight digits. In practice, an eight-place decimal will seldom be encountered. There are some inexpensive calculators that can handle 13 decimal places.

6.5.4.1 Sample Set B

Use a calculator to nd each sum or dierence.

Example 6.28

42.0638 + 126.551

Display Reads

Type 42.0638 42.0638

Press +

42.0638

Type 126.551 126.551

Press =

168.6148

Table 6.11

Available for free at Connexions <http://cnx.org/content/col10615/1.4>

358

CHAPTER 6. DECIMALS

The sum is 168.6148.

Example 6.29

Find the dierence between 305.0627 and 14.29667.

Display Reads

Type 305.0627 305.0627

Press

305.0627

Type 14.29667 14.29667

Press =

290.76603

Table 6.12

The dierence is 290.76603

Example 6.30

51.07 + 3,891.001786

Since 3,891.001786 contains more than eight digits, we will be unable to use an eight-digit display calculator to perform this addition. We can, however, nd the sum by hand.

51.070000

3891.001786

3942.071786

The sum is 3,942.071786.

6.5.4.2 Practice Set B

Use a calculator to perform each operation.

Exercise 6.5.7

(Solution on p. 407.)

4.286 + 8.97

Exercise 6.5.8

(Solution on p. 407.)

452.0092 − 392.558

Exercise 6.5.9

(Solution on p. 407.)

Find the sum of 0.095 and 0.001862

Exercise 6.5.10

(Solution on p. 407.)

Find the dierence between 0.5 and 0.025

Exercise 6.5.11

(Solution on p. 407.)

Find the sum of 2,776.00019 and 2,009.00012.

Available for free at Connexions <http://cnx.org/content/col10615/1.4>

359

6.5.5 Exercises

For the following 15 problems, perform each addition or subtraction. Use a calculator to check each result.

Exercise 6.5.12

(Solution on p. 407.)

1.84 + 7.11

Exercise 6.5.13

15.015 − 6.527

Exercise 6.5.14

(Solution on p. 407.)

11.842 + 28.004

Exercise 6.5.15

3.16 − 2.52

Exercise 6.5.16

(Solution on p. 407.)

3.55267 + 8.19664

Exercise 6.5.17

0.9162 − 0.0872

Exercise 6.5.18

(Solution on p. 408.)

65.512 − 8.3005

Exercise 6.5.19

761.0808 − 53.198

Exercise 6.5.20

(Solution on p. 408.)

4.305 + 2.119 − 3.817

Exercise 6.5.21

19.1161 + 27.8014 + 39.3161

Exercise 6.5.22

(Solution on p. 408.)

0.41276 − 0.0018 − 0.00011

Exercise 6.5.23

2.181 + 6.05 + 1.167 + 8.101

Exercise 6.5.24

(Solution on p. 408.)

1.0031+6.013106+0.00018+0.0092+2.11

Exercise 6.5.25

27 + 42 + 9.16 − 0.1761 + 81.6

Exercise 6.5.26

(Solution on p. 408.)

10.28 + 11.111 + 0.86 + 5.1

For the following 10 problems, solve as directed. A calculator may be useful.

Exercise 6.5.27

Add 6.1121 and 4.916 and round to 2 decimal places.

Exercise 6.5.28

(Solution on p. 408.)

Add 21.66418 and 18.00184 and round to 4 decimal places.

Exercise 6.5.29

Subtract 5.2121 from 9.6341 and round to 1 decimal place.

Exercise 6.5.30

(Solution on p. 408.)

Subtract 0.918 from 12.006 and round to 2 decimal places.

Exercise 6.5.31

Subtract 7.01884 from the sum of 13.11848 and 2.108 and round to 4 decimal places.

Available for free at Connexions <http://cnx.org/content/col10615/1.4>

360

CHAPTER 6. DECIMALS

Exercise 6.5.32

(Solution on p. 408.)

A checking account has a balance of $42.51. A check is written for $19.28. What is the new balance?

Exercise 6.5.33

A checking account has a balance of $82.97. One check is written for $6.49 and another for $39.95.

What is the new balance?

Exercise 6.5.34

(Solution on p. 408.)

A person buys $4.29 worth of hamburger and pays for it with a $10 bill. How much change does this person get?

Exercise 6.5.35

A man buys $6.43 worth of stationary and pays for it with a $20 bill. After receiving his change, he realizes he forgot to buy a pen. If the total price of the pen is $2.12, and he buys it, how much of the $20 bill is left?

Exercise 6.5.36

(Solution on p. 408.)

A woman starts recording a movie on her video cassette recorder with the tape counter set at 21.93. The movie runs 847.44 tape counter units. What is the nal tape counter reading?

6.5.5.1 Exercises for Review

Exercise 6.5.37

(Section 1.6) Find the dierence between 11,206 and 10,884.

Exercise 6.5.38

(Solution on p. 408.)

(Section 2.2) Find the product, 820 · 10,000.

Exercise 6.5.39

(Section 3.3) Find the value of 121 − 25 + 82 + 16 ÷ 22 .

Exercise 6.5.40

(Solution on p. 408.)

(Section 4.6) Find the value of 81 · 36

.

3

75 ÷ 2 25

Exercise 6.5.41

(Section 6.4) Round 1.08196 to the nearest hundredth.

6.6 Multiplication of Decimals6

6.6.1 Section Overview

• The Logic Behind the Method

• The Method of Multiplying Decimals

• Calculators

• Multiplying Decimals By Powers of 10

• Multiplication in Terms of Of

6This content is available online at <http://cnx.org/content/m34963/1.2/>.

Available for free at Connexions <http://cnx.org/content/col10615/1.4>

Image 298

Image 299

361

6.6.2 The Logic Behind the Method

Consider the product of 3.2 and 1.46. Changing each decimal to a fraction, we have (3.2) (1.46) =

3 2

10 · 1 46

100

=

32

10 · 146

100

=

32·146

10·100

=

4672

1000

=

4 672

1000

=

four and six hundred seventy-two thousandths

=

4.672

Thus, (3.2) (1.46) = 4.672.

Notice that the factor

3.2 has 1 decimal place,

1.46 has 2 decimal places, }1 + 2 = 3

and the product

4.672 has 3 decimal places.

Using this observation, we can suggest that the sum of the number of decimal places in the factors equals the number of decimal places in the product.

6.6.3 The Method of Multiplying Decimals

Method of Multiplying Decimals

To multiply decimals,

1. Multiply the numbers as if they were whole numbers.

2. Find the sum of the number of decimal places in the factors.

3. The number of decimal places in the product is the sum found in step 2.

6.6.3.1 Sample Set A

Find the following products.

Example 6.31

6.5 · 4.3

Available for free at Connexions <http://cnx.org/content/col10615/1.4>

Image 300

Image 301

Image 302

362

CHAPTER 6. DECIMALS

Thus, 6.5 · 4.3 = 27.95.

Example 6.32

23.4 · 1.96

Thus, 23.4 · 1.96 = 45.864.

Example 6.33

Find the product of 0.251 and 0.00113 and round to three decimal places.

Now, rounding to three decimal places, we get

6.6.3.2 Practice Set A

Find the following products.

Exercise 6.6.1

(Solution on p. 408.)

5.3 · 8.6

Exercise 6.6.2

(Solution on p. 408.)

2.12 · 4.9

Exercise 6.6.3

(Solution on p. 408.)

1.054 · 0.16

Exercise 6.6.4

(Solution on p. 408.)

0.00031 · 0.002

Exercise 6.6.5

(Solution on p. 408.)

Find the product of 2.33 and 4.01 and round to one decimal place.

Exercise 6.6.6

(Solution on p. 408.)

10 · 5.394

Exercise 6.6.7

(Solution on p. 408.)

100 · 5.394

Exercise 6.6.8

(Solution on p. 408.)

1000 · 5.394

Exercise 6.6.9

(Solution on p. 408.)

10,000 · 5.394

Available for free at Connexions <http://cnx.org/content/col10615/1.4>

363

6.6.4 Calculators

Calculators can be used to nd products of decimal numbers. However, a calculator that has only an eight-digit display may not be able to handle numbers or products that result in more than eight digits. But there are plenty of inexpensive ($50 – $75) calculators with more than eight-digit displays.

6.6.4.1 Sample Set B

Find the following products, if possible, using a calculator.

Example 6.34

2.58 · 8.61

Display Reads

Type 2.58 2.58

Press ×

2.58

Type 8.61 8.61

Press =

22.2138

Table 6.13

The product is 22.2138.

Example 6.35

0.006 · 0.0042

Display Reads

Type .006

.006

Press ×

.006

Type .0042 0.0042

Press =

0.0000252

Table 6.14

We know that there will be seven decimal places in the product (since 3 + 4 = 7). Since the display shows 7 decimal places, we can assume the product is correct. Thus, the product is 0.0000252.

Example 6.36

0.0026 · 0.11976

Since we expect 4 + 5 = 9 decimal places in the product, we know that an eight-digit display calculator will not be able to provide us with the exact value. To obtain the exact value, we must use “hand technology.” Suppose, however, that we agree to round o this product to three decimal places. We then need only four decimal places on the display.

Display Reads

Type .0026

.0026

Press ×

.0026

Type .11976 0.11976

Press =

0.0003114

Available for free at Connexions <http://cnx.org/content/col10615/1.4>

Image 303

364

CHAPTER 6. DECIMALS

Table 6.15

Rounding 0.0003114 to three decimal places we get 0.000. Thus, 0.0026 · 0.11976 = 0.000 to three decimal places.

6.6.4.2 Practice Set B

Use a calculator to nd each product. If the calculator will not provide the exact product, round the result to four decimal places.

Exercise 6.6.10

(Solution on p. 408.)

5.126 · 4.08

Exercise 6.6.11

(Solution on p. 408.)

0.00165 · 0.04

Exercise 6.6.12

(Solution on p. 408.)

0.5598 · 0.4281

Exercise 6.6.13

(Solution on p. 408.)

0.000002 · 0.06

6.6.5 Multiplying Decimals by Powers of 10

There is an interesting feature of multiplying decimals by powers of 10. Consider the following multiplications.

Multiplication

Number of Zeros in the Power of Number of Positions the Decimal

10

Point Has Been Moved to the

Right

10 · 8.315274 = 83.15274

1

1

100 · 8.315274 = 831.5274

2

2

1, 000 · 8.315274 = 8, 315.274

3

3

10, 000 · 8.315274 = 83, 152.74

4

4

Table 6.16

Multiplying a Decimal by a Power of 10

To multiply a decimal by a power of 10, move the decimal place to the right of its current position as many places as there are zeros in the power of 10. Add zeros if necessary.

6.6.5.1 Sample Set C

Find the following products.

Example 6.37

100 · 34.876. Since there are 2 zeros in 100, Move the decimal point in 34.876 two places to the right.

Available for free at Connexions <http://cnx.org/content/col10615/1.4>

Image 304

Image 305

Image 306

Image 307

365

Example 6.38

1, 000 · 4.8058. Since there are 3 zeros in 1,000, move the decimal point in 4.8058 three places to the right.

Example 6.39

10, 000 · 56.82. Since there are 4 zeros in 10,000, move the decimal point in 56.82 four places to the right. We will have to add two zeros in order to obtain the four places.

Since there is no fractional part, we can drop the decimal point.

Example 6.40

Example 6.41

6.6.5.2 Practice Set C

Find the following products.

Exercise 6.6.14

(Solution on p. 409.)

100 · 4.27

Exercise 6.6.15

(Solution on p. 409.)

10,000 · 16.52187

Exercise 6.6.16

(Solution on p. 409.)

(10) (0.0188)

Exercise 6.6.17

(Solution on p. 409.)

(10,000,000,000) (52.7)

6.6.6 Multiplication in Terms of Of

Recalling that the word “of” translates to the arithmetic operation of multiplication, let’s observe the following multiplications.

Available for free at Connexions <http://cnx.org/content/col10615/1.4>

366

CHAPTER 6. DECIMALS

6.6.6.1 Sample Set D

Example 6.42

Find 4.1 of 3.8.

Translating “of” to “×”, we get

4.1

×3.8

328

123

15.58

Thus, 4.1 of 3.8 is 15.58.

Example 6.43

Find 0.95 of the sum of 2.6 and 0.8.

We rst nd the sum of 2.6 and 0.8.

2.6

+0.8

3.4

Now nd 0.95 of 3.4

3.4

×0.95

170

306

3.230

Thus, 0.95 of (2.6 + 0.8) is 3.230.

6.6.6.2 Practice Set D

Exercise 6.6.18

(Solution on p. 409.)

Find 2.8 of 6.4.

Exercise 6.6.19

(Solution on p. 409.)

Find 0.1 of 1.3.

Exercise 6.6.20

(Solution on p. 409.)

Find 1.01 of 3.6.

Exercise 6.6.21

(Solution on p. 409.)

Find 0.004 of 0.0009.

Exercise 6.6.22

(Solution on p. 409.)

Find 0.83 of 12.

Exercise 6.6.23

(Solution on p. 409.)

Find 1.1 of the sum of 8.6 and 4.2.

Available for free at Connexions <http://cnx.org/content/col10615/1.4>

367

6.6.7 Exercises

For the following 30 problems, nd each product and check each result with a calculator.

Exercise 6.6.24

(Solution on p. 409.)

3.4 · 9.2

Exercise 6.6.25

4.5 · 6.1

Exercise 6.6.26

(Solution on p. 409.)

8.0 · 5.9

Exercise 6.6.27

6.1 · 7

Exercise 6.6.28

(Solution on p. 409.)

(0.1) (1.52)

Exercise 6.6.29

(1.99) (0.05)

Exercise 6.6.30

(Solution on p. 409.)

(12.52) (0.37)

Exercise 6.6.31

(5.116) (1.21)

Exercise 6.6.32

(Solution on p. 409.)

(31.82) (0.1)

Exercise 6.6.33

(16.527) (9.16)

Exercise 6.6.34

(Solution on p. 409.)

0.0021 · 0.013

Exercise 6.6.35

1.0037 · 1.00037

Exercise 6.6.36

(Solution on p. 409.)

(1.6) (1.6)

Exercise 6.6.37

(4.2) (4.2)

Exercise 6.6.38

(Solution on p. 409.)

0.9 · 0.9

Exercise 6.6.39

1.11 · 1.11

Exercise 6.6.40

(Solution on p. 409.)

6.815 · 4.3

Exercise 6.6.41

9.0168 · 1.2

Exercise 6.6.42

(Solution on p. 409.)

(3.5162) (0.0000003)

Exercise 6.6.43

(0.000001) (0.01)

Available for free at Connexions <http://cnx.org/content/col10615/1.4>

368

CHAPTER 6. DECIMALS

Exercise 6.6.44

(Solution on p. 409.)

(10) (4.96)

Exercise 6.6.45

(10) (36.17)

Exercise 6.6.46

(Solution on p. 409.)

10 · 421.8842

Exercise 6.6.47

10 · 8.0107

Exercise 6.6.48

(Solution on p. 409.)

100 · 0.19621

Exercise 6.6.49

100 · 0.779

Exercise 6.6.50

(Solution on p. 409.)

1000 · 3.596168

Exercise 6.6.51

1000 · 42.7125571

Exercise 6.6.52

(Solution on p. 409.)

1000 · 25.01

Exercise 6.6.53

100, 000 · 9.923

Exercise 6.6.54

(Solution on p. 410.)

(4.6) (6.17)

Actual product Tenths Hundreds Thousandths

Table 6.17

Exercise 6.6.55

(8.09) (7.1)

Actual product Tenths Hundreds Thousandths

Table 6.18

Exercise 6.6.56

(Solution on p. 410.)

(11.1106) (12.08)

Actual product Tenths Hundreds Thousandths

Table 6.19

Exercise 6.6.57

0.0083 · 1.090901

Available for free at Connexions <http://cnx.org/content/col10615/1.4>

369

Actual product Tenths Hundreds Thousandths

Table 6.20

Exercise 6.6.58

(Solution on p. 410.)

7 · 26.518

Actual product Tenths Hundreds Thousandths

Table 6.21

For the following 15 problems, perform the indicated operations

Exercise 6.6.59

Find 5.2 of 3.7.

Exercise 6.6.60

(Solution on p. 410.)

Find 12.03 of 10.1

Exercise 6.6.61

Find 16 of 1.04

Exercise 6.6.62

(Solution on p. 410.)

Find 12 of 0.1

Exercise 6.6.63

Find 0.09 of 0.003

Exercise 6.6.64

(Solution on p. 410.)

Find 1.02 of 0.9801

Exercise 6.6.65

Find 0.01 of the sum of 3.6 and 12.18

Exercise 6.6.66

(Solution on p. 410.)

Find 0.2 of the sum of 0.194 and 1.07

Exercise 6.6.67

Find the dierence of 6.1 of 2.7 and 2.7 of 4.03

Exercise 6.6.68

(Solution on p. 410.)

Find the dierence of 0.071 of 42 and 0.003 of 9.2

Exercise 6.6.69

If a person earns $8.55 an hour, how much does he earn in twenty-ve hundredths of an hour?

Exercise 6.6.70

(Solution on p. 410.)

A man buys 14 items at $1.16 each. What is the total cost?

Exercise 6.6.71

In the problem above, how much is the total cost if 0.065 sales tax is added?

Exercise 6.6.72

(Solution on p. 410.)

A river rafting trip is supposed to last for 10 days and each day 6 miles is to be rafted. On the third day a person falls out of the raft after only 2 of that day’s mileage. If this person gets discouraged 5

and quits, what fraction of the entire trip did he complete?

Available for free at Connexions <http://cnx.org/content/col10615/1.4>

370

CHAPTER 6. DECIMALS

Exercise 6.6.73

A woman starts the day with $42.28. She buys one item for $8.95 and another for $6.68. She then buys another item for sixty two-hundredths of the remaining amount. How much money does she have left?

Calculator Problems

For the following 10 problems, use a calculator to determine each product. If the calculator will not provide the exact product, round the results to ve decimal places.

Exercise 6.6.74

(Solution on p. 410.)

0.019 · 0.321

Exercise 6.6.75

0.261 · 1.96

Exercise 6.6.76

(Solution on p. 410.)

4.826 · 4.827

Exercise 6.6.77

(9.46)2

Exercise 6.6.78

(Solution on p. 410.)

(0.012)2

Exercise 6.6.79

0.00037 · 0.0065

Exercise 6.6.80

(Solution on p. 410.)

0.002 · 0.0009

Exercise 6.6.81

0.1286 · 0.7699

Exercise 6.6.82

(Solution on p. 410.)

0.01 · 0.00000471

Exercise 6.6.83

0.00198709 · 0.03

6.6.7.1 Exercises for Review

Exercise 6.6.84

(Solution on p. 410.)

(Section 2.3) Find the value, if it exists, of 0 ÷ 15.

Exercise 6.6.85

(Section 3.5) Find the greatest common factor of 210, 231, and 357.

Exercise 6.6.86

(Solution on p. 411.)

(Section 4.4) Reduce 280

2,156 to lowest terms.

Exercise 6.6.87

(Section 6.2) Write “fourteen and one hundred twenty-one ten-thousandths, using digits.”

Exercise 6.6.88

(Solution on p. 411.)

(Section 6.5) Subtract 6.882 from 8.661 and round the result to two decimal places.

Available for free at Connexions <http://cnx.org/content/col10615/1.4>

Image 308

371

6.7 Division of Decimals7

6.7.1 Section Overview

• The Logic Behind the Method

• A Method of Dividing a Decimal By a Nonzero Whole Number

• A Method of Dividing a Decimal by a Nonzero Decimal

• Dividing Decimals by Powers of 10

6.7.2 The Logic Behind the Method

As we have done with addition, subtraction, and multiplication of decimals, we will study a method of division of decimals by converting them to fractions, then we will make a general rule.

We will proceed by using this example: Divide 196.8 by 6.

32

6)196.8

18

16

12

4

We have, up to this point, divided 196.8 by 6 and have gotten a quotient of 32 with a remainder of 4. If we follow our intuition and bring down the .8, we have the division 4.8 ÷ 6.

4.8 ÷ 6 = 4 8

10 ÷ 6

=

48

10 ÷ 61

8

=

)48

10 · 1

)6

1

=

8

10

Thus, 4.8 ÷ 6 = .8.

Now, our intuition and experience with division direct us to place the .8 immediately to the right of 32.

From these observations, we suggest the following method of division.

7This content is available online at <http://cnx.org/content/m34968/1.2/>.

Available for free at Connexions <http://cnx.org/content/col10615/1.4>

Image 309

372

CHAPTER 6. DECIMALS

6.7.3 A Method of Dividing a Decimal by a Nonzero Whole Number

Method of Dividing a Decimal by a Nonzero Whole Number

To divide a decimal by a nonzero whole number:

1. Write a decimal point above the division line and directly over the decimal point of the dividend.

2. Proceed to divide as if both numbers were whole numbers.

3. If, in the quotient, the rst nonzero digit occurs to the right of the decimal point, but not in the tenths position, place a zero in each position between the decimal point and the rst nonzero digit of the quotient.

6.7.3.1 Sample Set A

Find the decimal representations of the following quotients.

Example 6.44

114.1 ÷ 7 = 7

16.3

7)114.1

7

44

42

2.1

2.1

0

Thus, 114.1 ÷ 7 = 16.3.

Check: If 114.1 ÷ 7 = 16.3, then 7 · 16.3 should equal 114.1.

4 2

16.3

7

114.1

True.

Example 6.45

0.02068 ÷ 4

Place zeros in the tenths and hundredths positions. (See Step 3.)

Thus, 0.02068 ÷ 4 = 0.00517.

Available for free at Connexions <http://cnx.org/content/col10615/1.4>

Image 310

373

6.7.3.2 Practice Set A

Find the following quotients.

Exercise 6.7.1

(Solution on p. 411.)

184.5 ÷ 3

Exercise 6.7.2

(Solution on p. 411.)

16.956 ÷ 9

Exercise 6.7.3

(Solution on p. 411.)

0.2964 ÷ 4

Exercise 6.7.4

(Solution on p. 411.)

0.000496 ÷ 8

6.7.4 A Method of Dividing a Decimal By a Nonzero Decimal

Now that we can divide decimals by nonzero whole numbers, we are in a position to divide decimals by a nonzero decimal. We will do so by converting a division by a decimal into a division by a whole number, a process with which we are already familiar. We’ll illustrate the method using this example: Divide 4.32 by 1.8.

Let’s look at this problem as 4 32

100 ÷ 1 8

10 .

4 32

4 32

100

100 ÷ 1 8

10

=

1 8

10

432

=

100

18

10

The divisor is 18

10 . We can convert 18

10 into a whole number if we multiply it by 10.

1

18

)10

10 · 10 = 18 ·

= 18

)10

1

1

But, we know from our experience with fractions, that if we multiply the denominator of a fraction by a nonzero whole number, we must multiply the numerator by that same nonzero whole number. Thus, when converting 18

10 to a whole number by multiplying it by 10, we must also multiply the numerator 432

100 by 10.

1

432

)10

100 · 10 = 432 ·

=

432·1 = 432

)100

1

10·1

10

10

=

43 210

=

43.2

We have converted the division 4.32 ÷ 1.8 into the division 43.2 ÷ 18, that is, 1.8)4.32 → 18)43.2

Notice what has occurred.

If we “move” the decimal point of the divisor one digit to the right, we must also “move” the decimal point of the dividend one place to the right. The word “move” actually indicates the process of multiplication by a power of 10.

Available for free at Connexions <http://cnx.org/content/col10615/1.4>

Image 311

Image 312

374

CHAPTER 6. DECIMALS

Method of Dividing a Decimal by a Decimal Number

To divide a decimal by a nonzero decimal,

1. Convert the divisor to a whole number by moving the decimal point to the position immediately to the right of the divisor’s last digit.

2. Move the decimal point of the dividend to the right the same number of digits it was moved in the divisor.

3. Set the decimal point in the quotient by placing a decimal point directly above the newly located decimal point in the dividend.

4. Divide as usual.

6.7.4.1 Sample Set B

Find the following quotients.

Example 6.46

32.66 ÷ 7.1

7.1)32.66

The divisor has one decimal place.

Move the decimal point of both the divisor and the dividend 1 place to the right.

Set the decimal point.

Divide as usual.

Thus, 32.66 ÷ 7.1 = 4.6.

Check: 32.66 ÷ 7.1 = 4.6 if 4.6 × 7.1 = 32.66

4.6

×7.1

46

322

32.66

True.

Example 6.47

1.0773 ÷ 0.513

The divisor has 3 decimal places.

Available for free at Connexions <http://cnx.org/content/col10615/1.4>

Image 313

375

Move the decimal point of both the divisor and the dividend 3 places to the right.

Set the decimal place and divide.

Thus, 1.0773 ÷ 0.513 = 2.1.

Checking by multiplying 2.1 and 0.513 will convince us that we have obtained the correct result.

(Try it.)

Example 6.48

12 ÷ 0.00032

0.00032)12.00000

The divisor has 5 decimal places.

Move the decimal point of both the divisor and the dividend 5 places to the right. We will need to add 5 zeros to 12.

Set the decimal place and divide.

This is now the same as the division of whole numbers.

37500.

32)1200000.

96

240

224

160

160

000

Checking assures us that 12 ÷ 0.00032 = 37, 500.

6.7.4.2 Practice Set B

Find the decimal representation of each quotient.

Exercise 6.7.5

(Solution on p. 411.)

9.176 ÷ 3.1

Exercise 6.7.6

(Solution on p. 411.)

5.0838 ÷ 1.11

Exercise 6.7.7

(Solution on p. 411.)

16 ÷ 0.0004

Exercise 6.7.8

(Solution on p. 411.)

8, 162.41 ÷ 10

Exercise 6.7.9

(Solution on p. 411.)

8, 162.41 ÷ 100

Exercise 6.7.10

(Solution on p. 411.)

8, 162.41 ÷ 1, 000

Available for free at Connexions <http://cnx.org/content/col10615/1.4>

376

CHAPTER 6. DECIMALS

Exercise 6.7.11

(Solution on p. 411.)

8, 162.41 ÷ 10, 000

6.7.5 Calculators

Calculators can be useful for nding quotients of decimal numbers. As we have seen with the other calculator operations, we can sometimes expect only approximate results. We are alerted to approximate results when the calculator display is lled with digits. We know it is possible that the operation may produce more digits than the calculator has the ability to show. For example, the multiplication 0.12345 × 0.4567

|

{z

}

|

{z

}

5 decimal

4 decimal

places

places

produces 5 + 4 = 9 decimal places. An eight-digit display calculator only has the ability to show eight digits, and an approximation results. The way to recognize a possible approximation is illustrated in problem 3 of the next sample set.

6.7.5.1 Sample Set C

Find each quotient using a calculator. If the result is an approximation, round to ve decimal places.

Example 6.49

12.596 ÷ 4.7

Display Reads

Type 12.596 12.596

Press ÷

12.596

Type 4.7

4.7

Press =

2.68

Table 6.22

Since the display is not lled, we expect this to be an accurate result.

Example 6.50

0.5696376 ÷ 0.00123

Display Reads

Type .5696376 0.5696376

Press ÷

0.5696376

Type .00123

0.00123

Press =

463.12

Table 6.23

Since the display is not lled, we expect this result to be accurate.

Example 6.51

0.8215199 ÷ 4.113

Available for free at Connexions <http://cnx.org/content/col10615/1.4>

377

Display Reads

Type .8215199 0.8215199

Press ÷

0.8215199

Type 4.113

4.113

Press =

0.1997373

Table 6.24

There are EIGHT DIGITS DISPLAY FILLED! BE AWARE OF POSSIBLE APPROXI-

MATIONS.

3

We can check for a possible approximation in the following way. Since the division 4)12 can be checked by multiplying 4 and 3, we can check our division by performing the multiplication 4.113 × 0.1997373

| {z }

|

{z

}

3 decimal

7 decimal

places

places

This multiplication produces 3 + 7 = 10 decimal digits. But our suspected quotient contains only 8

decimal digits. We conclude that the answer is an approximation. Then, rounding to ve decimal places, we get 0.19974.

6.7.5.2 Practice Set C

Find each quotient using a calculator. If the result is an approximation, round to four decimal places.

Exercise 6.7.12

(Solution on p. 411.)

42.49778 ÷ 14.261

Exercise 6.7.13

(Solution on p. 411.)

0.001455 ÷ 0.291

Exercise 6.7.14

(Solution on p. 411.)

7.459085 ÷ 2.1192

6.7.6 Dividing Decimals By Powers of 10

In problems 4 and 5 of Section 6.7.4.2 (Practice Set B), we found the decimal representations of 8, 162.41÷10

and 8, 162.41 ÷ 100. Let’s look at each of these again and then, from these observations, make a general statement regarding division of a decimal number by a power of 10.

Available for free at Connexions <http://cnx.org/content/col10615/1.4>

378

CHAPTER 6. DECIMALS

816.241

10)8162.410

80

16

10

62

60

24

20

41

40

10

10

0

Thus, 8, 162.41 ÷ 10 = 816.241.

Notice that the divisor 10 is composed of one 0 and that the quotient 816.241 can be obtained from the dividend 8,162.41 by moving the decimal point one place to the left.

81.6241

100)8162.4100

800

162

100

62 4

60 0

2 41

2 00

410

400

100

100

0

Thus, 8, 162.41 ÷ 100 = 81.6241.

Notice that the divisor 100 is composed of two 0’s and that the quotient 81.6241 can be obtained from the dividend by moving the decimal point two places to the left.

Using these observations, we can suggest the following method for dividing decimal numbers by powers of 10.

Available for free at Connexions <http://cnx.org/content/col10615/1.4>

Image 314

Image 315

379

Dividing a Decimal Fraction by a Power of 10

To divide a decimal fraction by a power of 10, move the decimal point of the decimal fraction to the left as many places as there are zeros in the power of 10. Add zeros if necessary.

6.7.6.1 Sample Set D

Find each quotient.

Example 6.52

9, 248.6 ÷ 100

Since there are 2 zeros in this power of 10, we move the decimal point 2 places to the left.

Example 6.53

3.28 ÷ 10, 000

Since there are 4 zeros in this power of 10, we move the decimal point 4 places to the left. To do so, we need to add three zeros.

6.7.6.2 Practice Set D

Find the decimal representation of each quotient.

Exercise 6.7.15

(Solution on p. 411.)

182.5 ÷ 10

Exercise 6.7.16

(Solution on p. 411.)

182.5 ÷ 100

Exercise 6.7.17

(Solution on p. 411.)

182.5 ÷ 1, 000

Exercise 6.7.18

(Solution on p. 411.)

182.5 ÷ 10, 000

Exercise 6.7.19

(Solution on p. 411.)

646.18 ÷ 100

Exercise 6.7.20

(Solution on p. 411.)

21.926 ÷ 1, 000

6.7.7 Exercises

For the following 30 problems, nd the decimal representation of each quotient. Use a calculator to check each result.

Exercise 6.7.21

(Solution on p. 411.)

4.8 ÷ 3

Exercise 6.7.22

16.8 ÷ 8

Exercise 6.7.23

(Solution on p. 411.)

18.5 ÷ 5

Available for free at Connexions <http://cnx.org/content/col10615/1.4>

380

CHAPTER 6. DECIMALS

Exercise 6.7.24

12.33 ÷ 3

Exercise 6.7.25

(Solution on p. 411.)

54.36 ÷ 9

Exercise 6.7.26

73.56 ÷ 12

Exercise 6.7.27

(Solution on p. 412.)

159.46 ÷ 17

Exercise 6.7.28

12.16 ÷ 64

Exercise 6.7.29

(Solution on p. 412.)

37.26 ÷ 81

Exercise 6.7.30

439.35 ÷ 435

Exercise 6.7.31

(Solution on p. 412.)

36.98 ÷ 4.3

Exercise 6.7.32

46.41 ÷ 9.1

Exercise 6.7.33

(Solution on p. 412.)

3.6 ÷ 1.5

Exercise 6.7.34

0.68 ÷ 1.7

Exercise 6.7.35

(Solution on p. 412.)

50.301 ÷ 8.1

Exercise 6.7.36

2.832 ÷ 0.4

Exercise 6.7.37

(Solution on p. 412.)

4.7524 ÷ 2.18

Exercise 6.7.38

16.2409 ÷ 4.03

Exercise 6.7.39

(Solution on p. 412.)

1.002001 ÷ 1.001

Exercise 6.7.40

25.050025 ÷ 5.005

Exercise 6.7.41

(Solution on p. 412.)

12.4 ÷ 3.1

Exercise 6.7.42

0.48 ÷ 0.08

Exercise 6.7.43

(Solution on p. 412.)

30.24 ÷ 2.16

Exercise 6.7.44

48.87 ÷ 0.87

Exercise 6.7.45

(Solution on p. 412.)

12.321 ÷ 0.111

Exercise 6.7.46

64, 351.006 ÷ 10

Available for free at Connexions <http://cnx.org/content/col10615/1.4>

381

Exercise 6.7.47

(Solution on p. 412.)

64, 351.006 ÷ 100

Exercise 6.7.48

64, 351.006 ÷ 1, 000

Exercise 6.7.49

(Solution on p. 412.)

64, 351.006 ÷ 1, 000, 000

Exercise 6.7.50

0.43 ÷ 100

For the following 5 problems, nd each quotient. Round to the specied position. A calculator may be used.

Exercise 6.7.51

(Solution on p. 412.)

11.2944 ÷ 6.24

Actual Quotient Tenths Hundredths Thousandths

Table 6.25

Exercise 6.7.52

45.32931 ÷ 9.01

Actual Quotient Tenths Hundredths Thousandths

Table 6.26

Exercise 6.7.53

(Solution on p. 412.)

3.18186 ÷ 0.66

Actual Quotient Tenths Hundredths Thousandths

Table 6.27

Exercise 6.7.54

4.3636 ÷ 4

Actual Quotient Tenths Hundredths Thousandths

Table 6.28

Exercise 6.7.55

(Solution on p. 412.)

0.00006318 ÷ 0.018

Actual Quotient Tenths Hundredths Thousandths

Available for free at Connexions <http://cnx.org/content/col10615/1.4>

382

CHAPTER 6. DECIMALS

Table 6.29

For the following 9 problems, nd each solution.

Exercise 6.7.56

Divide the product of 7.4 and 4.1 by 2.6.

Exercise 6.7.57

(Solution on p. 413.)

Divide the product of 11.01 and 0.003 by 2.56 and round to two decimal places.

Exercise 6.7.58

Divide the dierence of the products of 2.1 and 9.3, and 4.6 and 0.8 by 0.07 and round to one decimal place.

Exercise 6.7.59

(Solution on p. 413.)

A ring costing $567.08 is to be paid o in equal monthly payments of $46.84. In how many months will the ring be paid o?

Exercise 6.7.60

Six cans of cola cost $2.58. What is the price of one can?

Exercise 6.7.61

(Solution on p. 413.)

A family traveled 538.56 miles in their car in one day on their vacation. If their car used 19.8

gallons of gas, how many miles per gallon did it get?

Exercise 6.7.62

Three college students decide to rent an apartment together. The rent is $812.50 per month. How much must each person contribute toward the rent?

Exercise 6.7.63

(Solution on p. 413.)

A woman notices that on slow speed her video cassette recorder runs through 296.80 tape units in 10 minutes and at fast speed through 1098.16 tape units. How many times faster is fast speed than slow speed?

Exercise 6.7.64

A class of 34 rst semester business law students pay a total of $1,354.90, disregarding sales tax, for their law textbooks. What is the cost of each book?

Calculator Problems

For the following problems, use calculator to nd the quotients. If the result is approximate (see Sample Set C Example 6.51) round the result to three decimal places.

Exercise 6.7.65

(Solution on p. 413.)

3.8994 ÷ 2.01

Exercise 6.7.66

0.067444 ÷ 0.052

Exercise 6.7.67

(Solution on p. 413.)

14, 115.628 ÷ 484.74

Exercise 6.7.68

219, 709.36 ÷ 9941.6

Exercise 6.7.69

(Solution on p. 413.)

0.0852092 ÷ 0.49271

Exercise 6.7.70

2.4858225 ÷ 1.11611

Exercise 6.7.71

(Solution on p. 413.)

0.123432 ÷ 0.1111

Exercise 6.7.72

2.102838 ÷ 1.0305

Available for free at Connexions <http://cnx.org/content/col10615/1.4>

Image 316

383

6.7.7.1 Exercises for Review

Exercise 6.7.73

(Solution on p. 413.)

(Section 4.3) Convert 47 to an improper fraction.

8

Exercise 6.7.74

(Section 4.7) 2 of what number is 4?

7

5

Exercise 6.7.75

(Solution on p. 413.)

(Section 5.3) Find the sum. 415 + 710 + 3.

5

Exercise 6.7.76

(Section 6.4) Round 0.01628 to the nearest ten-thousandths.

Exercise 6.7.77

(Solution on p. 413.)

(Section 6.6) Find the product (2.06)(1.39)

6.8 Nonterminating Divisions8

6.8.1 Section Overview

• Nonterminating Divisions

• Denoting Nonterminating Quotients

6.8.2 Nonterminating Divisions

Let’s consider two divisions:

1. 9.8 ÷ 3.5

2. 4 ÷ 3

Terminating Divisions

Previously, we have considered divisions like example 1, which is an example of a terminating division. A terminating division is a division in which the quotient terminates after several divisions (the remainder is zero).

Exact Divisions

The quotient in this problem terminates in the tenths position. Terminating divisions are also called exact divisions.

Nonterminating Division

The division in example 2 is an example of a nonterminating division. A non-terminating division is a division that, regardless of how far we carry it out, always has a remainder.

8This content is available online at <http://cnx.org/content/m34969/1.2/>.

Available for free at Connexions <http://cnx.org/content/col10615/1.4>

Image 317

384

CHAPTER 6. DECIMALS

Repeating Decimal

We can see that the pattern in the brace is repeated endlessly. Such a decimal quotient is called a repeating decimal.

6.8.3 Denoting Nonterminating Quotients

We use three dots at the end of a number to indicate that a pattern repeats itself endlessly.

4 ÷ 3 = 1.333 . . .

Another way, aside from using three dots, of denoting an endlessly repeating pattern is to write a bar ( ) above the repeating sequence of digits.

4 ÷ 3 = 1.3

The bar indicates the repeated pattern of 3.

Repeating patterns in a division can be discovered in two ways:

1. As the division process progresses, should the remainder ever be the same as the dividend, it can be concluded that the division is nonterminating and that the pattern in the quotient repeats. This fact is illustrated in Example 6.54 of Section 6.8.3.1 (Sample Set A).

2. As the division process progresses, should the “product, dierence” pattern ever repeat two consecutive times, it can be concluded that the division is nonterminating and that the pattern in the quotient repeats. This fact is illustrated in Example 6.55 and 4 of Section 6.8.3.1 (Sample Set A).

6.8.3.1 Sample Set A

Carry out each division until the repeating pattern can be determined.

Example 6.54

100 ÷ 27

Available for free at Connexions <http://cnx.org/content/col10615/1.4>

Image 318

385

3.70370

27)100.00000

81

19 0

18 9

100

81

190

189

When the remainder is identical to the dividend, the division is nonterminating. This implies that the pattern in the quotient repeats.

100 ÷ 27 = 3.70370370 . . . The repeating block is 703.

100 ÷ 27 = 3.703

Example 6.55

1 ÷ 9

We see that this product, dierencepattern repeats. We can conclude that the division is nonterminating and that the quotient repeats.

1 ÷ 9 = 0.111 . . . The repeating block is 1.

1 ÷ 9 = 0.1

Example 6.56

Divide 2 by 11 and round to 3 decimal places.

Since we wish to round the quotient to three decimal places, we’ll carry out the division so that the quotient has four decimal places.

Available for free at Connexions <http://cnx.org/content/col10615/1.4>

Image 319

386

CHAPTER 6. DECIMALS

.1818

11)2.0000

1 1

90

88

20

11

90

The number .1818 rounded to three decimal places is .182. Thus, correct to three decimal places, 2 ÷ 11 = 0.182

Example 6.57

Divide 1 by 6.

We see that this product, dierence pattern repeats. We can conclude that the division is nonterminating and that the quotient repeats at the 6.

1 ÷ 6 = 0.16

6.8.3.2 Practice Set A

Carry out the following divisions until the repeating pattern can be determined.

Exercise 6.8.1

(Solution on p. 413.)

1 ÷ 3

Exercise 6.8.2

(Solution on p. 413.)

5 ÷ 6

Exercise 6.8.3

(Solution on p. 413.)

11 ÷ 9

Exercise 6.8.4

(Solution on p. 413.)

17 ÷ 9

Exercise 6.8.5

(Solution on p. 413.)

Divide 7 by 6 and round to 2 decimal places.

Exercise 6.8.6

(Solution on p. 413.)

Divide 400 by 11 and round to 4 decimal places.

Available for free at Connexions <http://cnx.org/content/col10615/1.4>

387

6.8.4 Exercises

For the following 20 problems, carry out each division until the repeating pattern is determined. If a repeating pattern is not apparent, round the quotient to three decimal places.

Exercise 6.8.7

(Solution on p. 413.)

4 ÷ 9

Exercise 6.8.8

8 ÷ 11

Exercise 6.8.9

(Solution on p. 413.)

4 ÷ 25

Exercise 6.8.10

5 ÷ 6

Exercise 6.8.11

(Solution on p. 413.)

1 ÷ 7

Exercise 6.8.12

3 ÷ 1.1

Exercise 6.8.13

(Solution on p. 413.)

20 ÷ 1.9

Exercise 6.8.14

10 ÷ 2.7

Exercise 6.8.15

(Solution on p. 413.)

1.11 ÷ 9.9

Exercise 6.8.16

8.08 ÷ 3.1

Exercise 6.8.17

(Solution on p. 413.)

51 ÷ 8.2

Exercise 6.8.18

0.213 ÷ 0.31

Exercise 6.8.19

(Solution on p. 413.)

0.009 ÷ 1.1

Exercise 6.8.20

6.03 ÷ 1.9

Exercise 6.8.21

(Solution on p. 413.)

0.518 ÷ 0.62

Exercise 6.8.22

1.55 ÷ 0.27

Exercise 6.8.23

(Solution on p. 413.)

0.333 ÷ 0.999

Exercise 6.8.24

0.444 ÷ 0.999

Exercise 6.8.25

(Solution on p. 414.)

0.555 ÷ 0.27

Exercise 6.8.26

3.8 ÷ 0.99

Calculator Problems

For the following 10 problems, use a calculator to perform each division.

Available for free at Connexions <http://cnx.org/content/col10615/1.4>

388

CHAPTER 6. DECIMALS

Exercise 6.8.27

(Solution on p. 414.)

7 ÷ 9

Exercise 6.8.28

8 ÷ 11

Exercise 6.8.29

(Solution on p. 414.)

14 ÷ 27

Exercise 6.8.30

1 ÷ 44

Exercise 6.8.31

(Solution on p. 414.)

2 ÷ 44

Exercise 6.8.32

0.7 ÷ 0.9 (Compare this with Exercise 6.8.27.)

Exercise 6.8.33

(Solution on p. 414.)

80 ÷ 110 (Compare this with Exercise 6.8.28.)

Exercise 6.8.34

0.0707 ÷ 0.7070

Exercise 6.8.35

(Solution on p. 414.)

0.1414 ÷ 0.2020

Exercise 6.8.36

1 ÷ 0.9999999

6.8.4.1 Exercise for Review

Exercise 6.8.37

(Solution on p. 414.)

(Section 1.2) In the number 411,105, how many ten thousands are there?

Exercise 6.8.38

(Section 2.3) Find the quotient, if it exists. 17 ÷ 0.

Exercise 6.8.39

(Solution on p. 414.)

(Section 3.6) Find the least common multiple of 45, 63, and 98.

Exercise 6.8.40

(Section 6.5) Subtract 8.01629 from 9.00187 and round the result to three decimal places.

Exercise 6.8.41

(Solution on p. 414.)

(Section 6.7) Find the quotient. 104.06 ÷ 12.1.

6.9 Converting a Fraction to a Decimal9

Now that we have studied and practiced dividing with decimals, we are also able to convert a fraction to a decimal. To do so we need only recall that a fraction bar can also be a division symbol. Thus, 3 not only 4

means “3 objects out of 4,” but can also mean “3 divided by 4.”

9This content is available online at <http://cnx.org/content/m34970/1.2/>.

Available for free at Connexions <http://cnx.org/content/col10615/1.4>

Image 320

389

6.9.1 Sample Set A

Convert the following fractions to decimals. If the division is nonterminating, round to two decimal places.

Example 6.58

3 . Divide 3 by 4.

4

.75

4)3.00

2 8

20

20

0

Thus, 3 = 0.75.

4

Example 6.59

1 Divide 1 by 5.

5

.2

5)1.0

1.0

0

Thus, 1 = 0.2

5

Example 6.60

5 . Divide 5 by 6.

6

5 = 0.833 · · · We are to round to two decimal places.

6

Thus, 5 = 0.83 to two decimal places.

6

Example 6.61

5 1 . Note that 5 1 = 5 + 1 .

8

8

8

Convert 1 to a decimal.

8

Available for free at Connexions <http://cnx.org/content/col10615/1.4>

390

CHAPTER 6. DECIMALS

.125

8)1.000

8

20

16

40

40

0

1 = .125

8

Thus, 51 = 5 + 1 = 5 + .125 = 5.125.

8

8

Example 6.62

0.16 1 . This is a complex decimal.

4

Note that the 6 is in the hundredths position.

The number 0.161 is read as “sixteen and one-fourth hundredths.”

4

13

16 1

16·4+1

65

0.16 1 =

)65

4

4

4

=

· 1

= 13·1

4

100 =

100

= 100

4

4·20 = 13

80

1

)100

20

Now, convert 13

80 to a decimal.

.1625

80)13.0000

8 0

5 00

4 80

200

160

400

400

0

Thus, 0.161 = 0.1625.

4

6.9.2 Practice Set A

Convert the following fractions and complex decimals to decimals (in which no proper fractions appear). If the divison is nonterminating, round to two decimal places.

Exercise 6.9.1

(Solution on p. 414.)

1

4

Available for free at Connexions <http://cnx.org/content/col10615/1.4>

391

Exercise 6.9.2

(Solution on p. 414.)

1

25

Exercise 6.9.3

(Solution on p. 414.)

1

6

Exercise 6.9.4

(Solution on p. 414.)

15

16

Exercise 6.9.5

(Solution on p. 414.)

0.9 12

Exercise 6.9.6

(Solution on p. 414.)

8.0126 38

6.9.3 Exercises

For the following 30 problems, convert each fraction or complex decimal number to a decimal (in which no proper fractions appear).

Exercise 6.9.7

(Solution on p. 414.)

1

2

Exercise 6.9.8

4

5

Exercise 6.9.9

(Solution on p. 414.)

7

8

Exercise 6.9.10

5

8

Exercise 6.9.11

(Solution on p. 414.)

3

5

Exercise 6.9.12

2

5

Exercise 6.9.13

(Solution on p. 414.)

1

25

Exercise 6.9.14

3

25

Exercise 6.9.15

(Solution on p. 414.)

1

20

Exercise 6.9.16

1

15

Exercise 6.9.17

(Solution on p. 414.)

1

50

Exercise 6.9.18

1

75

Exercise 6.9.19

(Solution on p. 414.)

1

3

Exercise 6.9.20

5

6

Exercise 6.9.21

(Solution on p. 414.)

3

16

Available for free at Connexions <http://cnx.org/content/col10615/1.4>

392

CHAPTER 6. DECIMALS

Exercise 6.9.22

9

16

Exercise 6.9.23

(Solution on p. 414.)

1

27

Exercise 6.9.24

5

27

Exercise 6.9.25

(Solution on p. 414.)

7

13

Exercise 6.9.26

9

14

Exercise 6.9.27

(Solution on p. 415.)

7 23

Exercise 6.9.28

8 5

16

Exercise 6.9.29

(Solution on p. 415.)

1 2

15

Exercise 6.9.30

65 522

Exercise 6.9.31

(Solution on p. 415.)

101 625

Exercise 6.9.32

0.1 12

Exercise 6.9.33

(Solution on p. 415.)

0.24 18

Exercise 6.9.34

5.66 23

Exercise 6.9.35

(Solution on p. 415.)

810.3106 516

Exercise 6.9.36

4.1 19

For the following 18 problems, convert each fraction to a decimal. Round to ve decimal places.

Exercise 6.9.37

(Solution on p. 415.)

1

9

Exercise 6.9.38

2

9

Exercise 6.9.39

(Solution on p. 415.)

3

9

Exercise 6.9.40

4

9

Exercise 6.9.41

(Solution on p. 415.)

5

9

Exercise 6.9.42

6

9

Exercise 6.9.43

(Solution on p. 415.)

7

9

Available for free at Connexions <http://cnx.org/content/col10615/1.4>

393

Exercise 6.9.44

8

9

Exercise 6.9.45

(Solution on p. 415.)

1

11

Exercise 6.9.46

2

11

Exercise 6.9.47

(Solution on p. 415.)

3

11

Exercise 6.9.48

4

11

Exercise 6.9.49

(Solution on p. 415.)

5

11

Exercise 6.9.50

6

11

Exercise 6.9.51

(Solution on p. 415.)

7

11

Exercise 6.9.52

8

11

Exercise 6.9.53

(Solution on p. 415.)

9

11

Exercise 6.9.54

10

11

Calculator Problems

For the following problems, use a calculator to convert each fraction to a decimal. If no repeating pattern seems to exist, round to four decimal places.

Exercise 6.9.55

(Solution on p. 415.)

16

125

Exercise 6.9.56

85

311

Exercise 6.9.57

(Solution on p. 415.)

192

197

Exercise 6.9.58

1

1469

Exercise 6.9.59

(Solution on p. 415.)

4

21,015

Exercise 6.9.60

81,426

106,001

Exercise 6.9.61

(Solution on p. 415.)

16,501

426

Available for free at Connexions <http://cnx.org/content/col10615/1.4>

394

CHAPTER 6. DECIMALS

6.9.3.1 Exercises for Review

Exercise 6.9.62

(Section 1.4) Round 2,105,106 to the nearest hundred thousand.

Exercise 6.9.63

(Solution on p. 415.)

(Section 4.7) 8 of what number is 3?

5

2

Exercise 6.9.64

(Section 5.5) Arrange 1 916, 15, and 1 7

8

12 in increasing order.

Exercise 6.9.65

(Solution on p. 415.)

(Section 6.3) Convert the complex decimal 3.65 to a fraction.

4

Exercise 6.9.66

(Section 6.8) Find the quotient. 30 ÷ 1.1.

6.10 Combinations of Operations with Decimals and Fractions10

Having considered operations with decimals and fractions, we now consider operations that involve both decimals and fractions.

6.10.1 Sample Set A

Perform the following operations.

Example 6.63

0.38· 1 . Convert both numbers to decimals or both numbers to fractions. We’ll convert to decimals.

4

.25

4)1.00

8

20

20

0

To convert 1 to a decimal, divide 1 by 4.

4

Now multiply 0.38 and .25.

1

4

.3 8

×.25

190

76

.0950

Thus, 0.38 · 1 = 0.095.

4

10This content is available online at <http://cnx.org/content/m34971/1.2/>.

Available for free at Connexions <http://cnx.org/content/col10615/1.4>

395

In the problems that follow, the conversions from fraction to decimal, or decimal to fraction, and some of the additions, subtraction, multiplications, and divisions will be left to you.

Example 6.64

1.85 + 3 · 4.1 Convert 3 to a decimal.

8

8

1.85 + 0.375 · 4.1 Multiply before adding.

1.85 + 1.5375 Now add.

3.3875

Example 6.65

5

4

13

− 0.28 Convert 0.28 to a fraction.

5

5

4

4

13

− 28

=

5

− 7

5

100

13 5

25

=

5

20

13 25 − 7

25

1

1

=

)5 · )13

)13

)25

1

5

=

1

5

Example 6.66

125

0.125 + 1 − 0.1211 =

1000

+ 1 − 0.1211

1 1

16

4

16

3

3

1

=

8

+ 1 − 0.1211

4

16

3

=

1 · 3 + 1 − 0.1211

8

4

16

=

3 + 1 − 0.1211

32

16

=

3 + 2 − 0.1211 = 5 − 0.1211

32

32

32

=

0.15625 − 0.1211

=

0.03515

Convert this to fraction form

=

3515

100,000

=

703

20,000

6.10.2 Practice Set A

Perform the following operations.

Exercise 6.10.1

(Solution on p. 415.)

3 + 1.6

5

Exercise 6.10.2

(Solution on p. 415.)

8.91 + 1 · 1.6

5

Exercise 6.10.3

(Solution on p. 415.)

1 9

16 6.12 + 7

25

Exercise 6.10.4

(Solution on p. 415.)

0.156 − 0.05

1 11

15

Available for free at Connexions <http://cnx.org/content/col10615/1.4>

396

CHAPTER 6. DECIMALS

6.10.3 Exercises

Exercise 6.10.5

(Solution on p. 415.)

3

10 + 0.7

Exercise 6.10.6

1 + 0.1

5

Exercise 6.10.7

(Solution on p. 416.)

5 − 0.513

8

Exercise 6.10.8

0.418 − 67

200

Exercise 6.10.9

(Solution on p. 416.)

0.22 · 14

Exercise 6.10.10

3 · 8.4

5

Exercise 6.10.11

(Solution on p. 416.)

1

25 · 3.19

Exercise 6.10.12

3

20 ÷ 0.05

Exercise 6.10.13

(Solution on p. 416.)

7

40 ÷ 0.25

Exercise 6.10.14

1 1

15 ÷ 0.9 · 0.12

Exercise 6.10.15

(Solution on p. 416.)

9.26 + 1 · 0.81

4

Exercise 6.10.16

0.588 + 1

40 · 0.24

Exercise 6.10.17

(Solution on p. 416.)

1

20 + 3.62 · 38

Exercise 6.10.18

7 + 0.15÷ 3

30

Exercise 6.10.19

(Solution on p. 416.)

15

16 ·

7

10 − 0.5

Exercise 6.10.20

0.2 · 7

20 + 1.1143

Exercise 6.10.21

(Solution on p. 416.)

3 · 0.875 + 1

4

8

Exercise 6.10.22

5.198 − 0.26 · 14

250 + 0.119

Exercise 6.10.23

(Solution on p. 416.)

0.5 1 + (0.3)2

4

Exercise 6.10.24

(1.4)2 − 1.6 12

Exercise 6.10.25

(Solution on p. 416.)

3 2 − 0.000625 + (1.1)2

8

Exercise 6.10.26

(0.6)2 · 1

20 − 1

25

Available for free at Connexions <http://cnx.org/content/col10615/1.4>

397

Exercise 6.10.27

(Solution on p. 416.)

1 2 − 0.125

2

Exercise 6.10.28

0.75 + 5

4 1

12

2

Exercise 6.10.29

(Solution on p. 416.)

0.375 − 1

2 1

33

16

Exercise 6.10.30

1 1

8 1 ·

4

3

2.25 + 9

25

Exercise 6.10.31

(Solution on p. 416.)

0.32

12

35

0.35

Exercise

6.10.32

49 −5 0.125

64

1.375

6.10.3.1 Exercises for Review

Exercise 6.10.33

(Solution on p. 416.)

(Section 2.5) Is 21,480 divisible by 3?

Exercise 6.10.34

(Section 3.2) Expand 144. Do not nd the actual value.

Exercise 6.10.35

(Solution on p. 416.)

(Section 3.3) Find the prime factorization of 15,400.

Exercise 6.10.36

(Section 6.3) Convert 8.016 to a fraction.

Exercise 6.10.37

(Solution on p. 416.)

(Section 6.9) Find the quotient. 16 ÷ 27.

Available for free at Connexions <http://cnx.org/content/col10615/1.4>

398

CHAPTER 6. DECIMALS

6.11 Summary of Key Concepts11

6.11.1 Summary of Key Concepts

Decimal Point (Section 6.2)

A decimal point is a point that separates the units digit from the tenths digit.

Decimal or Decimal Fraction (Section 6.2)

A decimal fraction is a fraction whose denominator is a power of ten.

Converting a Decimal to a Fraction (Section 6.3)

Decimals can be converted to fractions by saying the decimal number in words, then writing what was said.

Rounding Decimals (Section 6.4)

Decimals are rounded in much the same way whole numbers are rounded.

Addition and Subtraction of Decimals (Section 6.5)

To add or subtract decimals,

1. Align the numbers vertically so that the decimal points line up under each other and the corresponding decimal positions are in the same column.

2. Add or subtract the numbers as if they were whole numbers.

3. Place a decimal point in the resulting sum directly under the other decimal points.

Multiplication of Decimals (Section 6.6)

To multiply two decimals,

1. Multiply the numbers as if they were whole numbers.

2. Find the sum of the number of decimal places in the factors.

3. The number of decimal places in the product is the number found in step 2.

Multiplying Decimals by Powers of 10 (Section 6.6)

To multiply a decimal by a power of 10, move the decimal point to the right as many places as there are zeros in the power of ten. Add zeros if necessary.

Division of a Decimal by a Decimal (Section 6.7)

To divide a decimal by a nonzero decimal,

1. Convert the divisor to a whole number by moving the decimal point until it appears to the right of the divisor’s last digit.

2. Move the decimal point of the dividend to the right the same number of digits it was moved in the divisor.

3. Proceed to divide.

4. Locate the decimal in the answer by bringing it straight up from the dividend.

Dividing Decimals by Powers of 10 (Section 6.7)

To divide a decimal by a power of 10, move the decimal point to the left as many places as there are zeros in the power of ten. Add zeros if necessary.

Terminating Divisions (Section 6.8)

A terminating division is a division in which the quotient terminates after several divisions. Terminating divisions are also called exact divisions.

11This content is available online at <http://cnx.org/content/m34972/1.2/>.

Available for free at Connexions <http://cnx.org/content/col10615/1.4>

399

Nonterminating Divisions (Section 6.8)

A nonterminating division is a division that, regardless of how far it is carried out, always has a remainder.

Nonterminating divisions are also called nonexact divisions.

Converting Fractions to Decimals (Section 6.9)

A fraction can be converted to a decimal by dividing the numerator by the denominator.

6.12 Exercise Supplement12

6.12.1 Exercise Supplement

6.12.1.1 Reading and Writing Decimals (Section 6.2)

Exercise 6.12.1

(Solution on p. 416.)

The decimal digit that appears two places to the right of the decimal point is in the position.

Exercise 6.12.2

The decimal digit that appears four places to the right of the decimal point is in the position.

For problems 3-8, read each decimal by writing it in words.

Exercise 6.12.3

(Solution on p. 416.)

7.2

Exercise 6.12.4

8.105

Exercise 6.12.5

(Solution on p. 416.)

16.52

Exercise 6.12.6

5.9271

Exercise 6.12.7

(Solution on p. 416.)

0.005

Exercise 6.12.8

4.01701

For problems 9-13, write each decimal using digits.

Exercise 6.12.9

(Solution on p. 416.)

Nine and twelve-hundredths.

Exercise 6.12.10

Two and one hundred seventy-seven thousandths.

Exercise 6.12.11

(Solution on p. 416.)

Fifty-six and thirty-ve ten-thousandths.

Exercise 6.12.12

Four tenths.

Exercise 6.12.13

(Solution on p. 416.)

Four thousand eighty-one millionths.

12This content is available online at <http://cnx.org/content/m34978/1.2/>.

Available for free at Connexions <http://cnx.org/content/col10615/1.4>

400

CHAPTER 6. DECIMALS

6.12.1.2 Converting a Decimal to a Fraction (Section 6.3)

For problem 14-20, convert each decimal to a proper fraction or a mixed number.

Exercise 6.12.14

1.07

Exercise 6.12.15

(Solution on p. 416.)

58.63

Exercise 6.12.16

0.05

Exercise 6.12.17

(Solution on p. 416.)

0.14 23

Exercise 6.12.18

1.09 18

Exercise 6.12.19

(Solution on p. 417.)

4.01 1

27

Exercise 6.12.20

9.11 19

6.12.1.3 Rounding Decimals (Section 6.4)

For problems 21-25, round each decimal to the specied position.

Exercise 6.12.21

(Solution on p. 417.)

4.087 to the nearest hundredth.

Exercise 6.12.22

4.087 to the nearest tenth.

Exercise 6.12.23

(Solution on p. 417.)

16.5218 to the nearest one.

Exercise 6.12.24

817.42 to the nearest ten.

Exercise 6.12.25

(Solution on p. 417.)

0.9811602 to the nearest one.

6.12.1.4 Addition, Subtraction, Multiplication and Division of Decimals, and Nonterminating Divisions (Section 6.5,Section 6.6,Section 6.7,Section 6.8)

For problem 26-45, perform each operation and simplify.

Exercise 6.12.26

7.10 + 2.98

Exercise 6.12.27

(Solution on p. 417.)

14.007 − 5.061

Exercise 6.12.28

1.2 · 8.6

Exercise 6.12.29

(Solution on p. 417.)

41.8 · 0.19

Exercise 6.12.30

57.51 ÷ 2.7

Available for free at Connexions <http://cnx.org/content/col10615/1.4>

401

Exercise 6.12.31

(Solution on p. 417.)

0.54003 ÷ 18.001

Exercise 6.12.32

32,051.3585 ÷ 23, 006.9999

Exercise 6.12.33

(Solution on p. 417.)

100 · 1, 816.001

Exercise 6.12.34

1, 000 · 1, 816.001

Exercise 6.12.35

(Solution on p. 417.)

10.000 · 0.14

Exercise 6.12.36

0.135888 ÷ 16.986

Exercise 6.12.37

(Solution on p. 417.)

150.79 ÷ 100

Exercise 6.12.38

4.119 ÷ 10, 000

Exercise 6.12.39

(Solution on p. 417.)

42.7 ÷ 18

Exercise 6.12.40

6.9 ÷ 12

Exercise 6.12.41

(Solution on p. 417.)

0.014 ÷ 47.6. Round to three decimal places.

Exercise 6.12.42

8.8 ÷ 19. Round to one decimal place.

Exercise 6.12.43

(Solution on p. 417.)

1.1 ÷ 9

Exercise 6.12.44

1.1 ÷ 9.9

Exercise 6.12.45

(Solution on p. 417.)

30 ÷ 11.1

6.12.1.5 Converting a Fraction to a Decimal (Section 6.9)

For problems 46-55, convert each fraction to a decimal.

Exercise 6.12.46

3

8

Exercise 6.12.47

(Solution on p. 417.)

43

100

Exercise 6.12.48

82

1000

Exercise 6.12.49

(Solution on p. 417.)

9 47

Exercise 6.12.50

8 5

16

Available for free at Connexions <http://cnx.org/content/col10615/1.4>

402

CHAPTER 6. DECIMALS

Exercise 6.12.51

(Solution on p. 417.)

1.3 13

Exercise 6.12.52

25.623

Exercise 6.12.53

(Solution on p. 417.)

125.12518

Exercise 6.12.54

9.11 19

Exercise 6.12.55

(Solution on p. 417.)

0.0 56

6.12.1.6 Combinations of Operations with Decimals and Fractions (Section 6.10) For problems 56-62, perform each operation.

Exercise 6.12.56

5 · 0.25

8

Exercise 6.12.57

(Solution on p. 417.)

3

16 · 1.36

Exercise 6.12.58

3 · 1 + 1.75

5

2

Exercise 6.12.59

(Solution on p. 417.)

7 · 5 + 0.30

2

4

Exercise 6.12.60

19.375 ÷ 4.375 − 1 1

16

Exercise 6.12.61

(Solution on p. 417.)

15

602 · 2.6 + 3 14

Exercise 6.12.62

4 13

18 ÷ 5 3

14 + 3 5

21

6.13 Prociency Exam13

6.13.1 Prociency Exam

Exercise 6.13.1

(Solution on p. 417.)

(Section 6.2) The decimal digit that appears three places to the right of the decimal point is in the

position.

Exercise 6.13.2

(Solution on p. 417.)

(Section 6.2) Write, using words, 15.036.

Exercise 6.13.3

(Solution on p. 417.)

(Section 6.2) Write eighty-one and twelve hundredths using digits. 81.12

Exercise 6.13.4

(Solution on p. 418.)

(Section 6.2) Write three thousand seventeen millionths using digits.

Exercise 6.13.5

(Solution on p. 418.)

(Section 6.3) Convert 0.78 to a fraction. Reduce.

13This content is available online at <http://cnx.org/content/m34979/1.2/>.

Available for free at Connexions <http://cnx.org/content/col10615/1.4>

403

Exercise 6.13.6

(Solution on p. 418.)

(Section 6.3) Convert 0.875 to a fraction. Reduce.

Exercise 6.13.7

(Solution on p. 418.)

(Section 6.4) Round 4.8063 to the nearest tenth.

Exercise 6.13.8

(Solution on p. 418.)

(Section 6.4) Round 187.51 to the nearest hundred.

Exercise 6.13.9

(Solution on p. 418.)

(Section 6.4) Round 0.0652 to the nearest hundredth.

For problems 10-20, perform each operation.

Exercise 6.13.10

(Solution on p. 418.)

(Section 6.5) 15.026 + 5.971

Exercise 6.13.11

(Solution on p. 418.)

(Section 6.5) 72.15 − 26.585

Exercise 6.13.12

(Solution on p. 418.)

(Section 6.6) 16.2 · 4.8

Exercise 6.13.13

(Solution on p. 418.)

(Section 6.6) 10, 000 · 0.016

Exercise 6.13.14

(Solution on p. 418.)

(Section 6.7) 44.64 ÷ 18.6

Exercise 6.13.15

(Solution on p. 418.)

(Section 6.7) 0.21387 ÷ 0.19

Exercise 6.13.16

(Solution on p. 418.)

(Section 6.8) 0.27 − 311

Exercise 6.13.17

(Solution on p. 418.)

(Section 6.9) Convert 6 211 to a decimal.

Exercise 6.13.18

(Solution on p. 418.)

(Section 6.9) Convert 0.5 9

16 to a decimal.

Exercise 6.13.19

(Solution on p. 418.)

(Section 6.10) 31 + 2.325

8

Exercise 6.13.20

(Solution on p. 418.)

(Section 6.10) 3 × 0.5625

8

Available for free at Connexions <http://cnx.org/content/col10615/1.4>

404

CHAPTER 6. DECIMALS

Solutions to Exercises in Chapter 6

Solution to Exercise 6.2.1 (p. 343)

twelve and nine tenths

Solution to Exercise 6.2.2 (p. 343)

four and eighty-six hundredths

Solution to Exercise 6.2.3 (p. 343)

seven and two hundred thousandths

Solution to Exercise 6.2.4 (p. 343)

thirty thousand four hundred ve millionths

Solution to Exercise 6.2.5 (p. 344)

306.49

Solution to Exercise 6.2.6 (p. 344)

9.004

Solution to Exercise 6.2.7 (p. 344)

0.000061

Solution to Exercise 6.2.8 (p. 344)

Tenths; hundredths, thousandths

Solution to Exercise 6.2.10 (p. 344)

Hundred thousandths; ten millionths

Solution to Exercise 6.2.12 (p. 344)

eight and one tenth

Solution to Exercise 6.2.14 (p. 344)

fty-ve and six hundredths

Solution to Exercise 6.2.16 (p. 344)

one and nine hundred four thousandths

Solution to Exercise 6.2.18 (p. 345)

3.20

Solution to Exercise 6.2.20 (p. 345)

1.8

Solution to Exercise 6.2.22 (p. 345)

511.004

Solution to Exercise 6.2.24 (p. 345)

0.947

Solution to Exercise 6.2.26 (p. 345)

0.00071

Solution to Exercise 6.2.28 (p. 345)

seventy-ve hundredths

Solution to Exercise 6.2.30 (p. 345)

four tenths

Solution to Exercise 6.2.32 (p. 345)

sixteen hundredths

Solution to Exercise 6.2.34 (p. 345)

one thousand eight hundred seventy-ve ten thousandths

Solution to Exercise 6.2.36 (p. 345)

fty-ve hundredths

Solution to Exercise 6.2.38 (p. 346)

2610

Solution to Exercise 6.2.40 (p. 346)

12

Solution to Exercise 6.2.42 (p. 346)

10

or 11

9

9

Available for free at Connexions <http://cnx.org/content/col10615/1.4>

405

Solution to Exercise 6.3.1 (p. 347)

1621

25

Solution to Exercise 6.3.2 (p. 347)

513

1,000

Solution to Exercise 6.3.3 (p. 347)

6, 646 107

10,000

Solution to Exercise 6.3.4 (p. 347)

1 1

10

Solution to Exercise 6.3.5 (p. 348)

7

8

Solution to Exercise 6.3.6 (p. 348)

31

250

Solution to Exercise 6.3.7 (p. 348)

6 7

1,200

Solution to Exercise 6.3.8 (p. 348)

18 217

Solution to Exercise 6.3.9 (p. 348)

7

10

Solution to Exercise 6.3.11 (p. 348)

53

100

Solution to Exercise 6.3.13 (p. 348)

219

1,000

Solution to Exercise 6.3.15 (p. 348)

4 45

Solution to Exercise 6.3.17 (p. 348)

16 325

Solution to Exercise 6.3.19 (p. 349)

6 1

2,000

Solution to Exercise 6.3.21 (p. 349)

1618

Solution to Exercise 6.3.23 (p. 349)

3 1

25

Solution to Exercise 6.3.25 (p. 349)

8 9

40

Solution to Exercise 6.3.27 (p. 349)

9 19,999

20,000

Solution to Exercise 6.3.29 (p. 349)

3

4

Solution to Exercise 6.3.31 (p. 349)

2 13

80

Solution to Exercise 6.3.33 (p. 349)

14 337

3,000

Solution to Exercise 6.3.35 (p. 349)

1 129

320

Solution to Exercise 6.3.37 (p. 349)

2 1

25

Solution to Exercise 6.3.39 (p. 350)

14

Solution to Exercise 6.3.41 (p. 350)

9

10

Available for free at Connexions <http://cnx.org/content/col10615/1.4>

406

CHAPTER 6. DECIMALS

Solution to Exercise 6.3.43 (p. 350)

thousandths

Solution to Exercise 6.4.1 (p. 351)

4.82

Solution to Exercise 6.4.2 (p. 351)

0.3593

Solution to Exercise 6.4.3 (p. 351)

82

Solution to Exercise 6.4.4 (p. 351)

800

Solution to Exercise 6.4.5 (p. 351)

43.994

Solution to Exercise 6.4.6 (p. 351)

105.0200

Solution to Exercise 6.4.7 (p. 351)

100.00

Solution to Exercise 6.4.8 (p. 352)

Tenth Hundredth Thousandth Ten Thousandth

20.0

20.01

20.011

20.0107

Table 6.30

Solution to Exercise 6.4.10 (p. 352)

Tenth Hundredth Thousandth Ten Thousandth

531.2

531.22

531.219

531.2188

Table 6.31

Solution to Exercise 6.4.12 (p. 352)

Tenth Hundredth Thousandth Ten Thousandth

2.0

2.00

2.000

2.0000

Table 6.32

Solution to Exercise 6.4.14 (p. 353)

Tenth Hundredth Thousandth Ten Thousandth

0.0

0.00

0.000

0.0000

Table 6.33

Solution to Exercise 6.4.16 (p. 353)

Available for free at Connexions <http://cnx.org/content/col10615/1.4>

407

Tenth Hundredth Thousandth Ten Thousandth

9.2

9.19

9.192

9.1919

Table 6.34

Solution to Exercise 6.4.18 (p. 353)

18.417

Solution to Exercise 6.4.20 (p. 353)

18.41681

Solution to Exercise 6.4.22 (p. 354)

18.42

Solution to Exercise 6.4.24 (p. 354)

0.1

Solution to Exercise 6.4.26 (p. 354)

0.83607

Solution to Exercise 6.4.28 (p. 354)

5.333

Solution to Exercise 6.4.30 (p. 354)

Ten million

Solution to Exercise 6.4.32 (p. 354)

256

Solution to Exercise 6.4.34 (p. 354)

3 4

25

Solution to Exercise 6.5.1 (p. 357)

6.179

Solution to Exercise 6.5.2 (p. 357)

9.646

Solution to Exercise 6.5.3 (p. 357)

1.6044

Solution to Exercise 6.5.4 (p. 357)

0.0065

Solution to Exercise 6.5.5 (p. 357)

25,777.22

Solution to Exercise 6.5.6 (p. 357)

25.69

Solution to Exercise 6.5.7 (p. 358)

13.256

Solution to Exercise 6.5.8 (p. 358)

59.4512

Solution to Exercise 6.5.9 (p. 358)

0.096862

Solution to Exercise 6.5.10 (p. 358)

0.475

Solution to Exercise 6.5.11 (p. 358)

Since each number contains more than eight digits, using some calculators may not be helpful. Adding these by hand technology, we get 4,785.00031

Solution to Exercise 6.5.12 (p. 359)

8.95

Solution to Exercise 6.5.14 (p. 359)

39.846

Available for free at Connexions <http://cnx.org/content/col10615/1.4>

408

CHAPTER 6. DECIMALS

Solution to Exercise 6.5.16 (p. 359)

11.74931

Solution to Exercise 6.5.18 (p. 359)

57.2115

Solution to Exercise 6.5.20 (p. 359)

2.607

Solution to Exercise 6.5.22 (p. 359)

0.41085

Solution to Exercise 6.5.24 (p. 359)

9.135586

Solution to Exercise 6.5.26 (p. 359)

27.351

Solution to Exercise 6.5.28 (p. 359)

39.6660

Solution to Exercise 6.5.30 (p. 359)

11.09

Solution to Exercise 6.5.32 (p. 360)

$23.23

Solution to Exercise 6.5.34 (p. 360)

$5.71

Solution to Exercise 6.5.36 (p. 360)

869.37

Solution to Exercise 6.5.38 (p. 360)

8,200,000

Solution to Exercise 6.5.40 (p. 360)

20 = 5 or 22

9

3

9

Solution to Exercise 6.6.1 (p. 362)

45.58

Solution to Exercise 6.6.2 (p. 362)

10.388

Solution to Exercise 6.6.3 (p. 362)

0.16864

Solution to Exercise 6.6.4 (p. 362)

0.00000062

Solution to Exercise 6.6.5 (p. 362)

9.3

Solution to Exercise 6.6.6 (p. 362)

53.94

Solution to Exercise 6.6.7 (p. 362)

539.4

Solution to Exercise 6.6.8 (p. 362)

5,394

Solution to Exercise 6.6.9 (p. 362)

59,340

Solution to Exercise 6.6.10 (p. 364)

20.91408

Solution to Exercise 6.6.11 (p. 364)

0.000066

Solution to Exercise 6.6.12 (p. 364)

0.2397

Available for free at Connexions <http://cnx.org/content/col10615/1.4>

409

Solution to Exercise 6.6.13 (p. 364)

0.0000

Solution to Exercise 6.6.14 (p. 365)

427

Solution to Exercise 6.6.15 (p. 365)

165,218.7

Solution to Exercise 6.6.16 (p. 365)

0.188

Solution to Exercise 6.6.17 (p. 365)

527,000,000,000

Solution to Exercise 6.6.18 (p. 366)

17.92

Solution to Exercise 6.6.19 (p. 366)

0.13

Solution to Exercise 6.6.20 (p. 366)

3.636

Solution to Exercise 6.6.21 (p. 366)

0.0000036

Solution to Exercise 6.6.22 (p. 366)

9.96

Solution to Exercise 6.6.23 (p. 366)

14.08

Solution to Exercise 6.6.24 (p. 367)

31.28

Solution to Exercise 6.6.26 (p. 367)

47.20

Solution to Exercise 6.6.28 (p. 367)

0.152

Solution to Exercise 6.6.30 (p. 367)

4.6324

Solution to Exercise 6.6.32 (p. 367)

3.182

Solution to Exercise 6.6.34 (p. 367)

0.0000273

Solution to Exercise 6.6.36 (p. 367)

2.56

Solution to Exercise 6.6.38 (p. 367)

0.81

Solution to Exercise 6.6.40 (p. 367)

29.3045

Solution to Exercise 6.6.42 (p. 367)

0.00000105486

Solution to Exercise 6.6.44 (p. 368)

49.6

Solution to Exercise 6.6.46 (p. 368)

4,218.842

Solution to Exercise 6.6.48 (p. 368)

19.621

Solution to Exercise 6.6.50 (p. 368)

3,596.168

Available for free at Connexions <http://cnx.org/content/col10615/1.4>

410

CHAPTER 6. DECIMALS

Solution to Exercise 6.6.52 (p. 368)

25,010

Solution to Exercise 6.6.54 (p. 368)

Actual product Tenths Hundreds Thousandths

28.382

28.4

28.38

28.382

Table 6.35

Solution to Exercise 6.6.56 (p. 368)

Actual product Tenths Hundreds Thousandths

134.216048

134.2

134.22

134.216

Table 6.36

Solution to Exercise 6.6.58 (p. 369)

Actual product Tenths Hundreds Thousandths

185.626

185.6

185.63

185.626

Table 6.37

Solution to Exercise 6.6.60 (p. 369)

121.503

Solution to Exercise 6.6.62 (p. 369)

1.2

Solution to Exercise 6.6.64 (p. 369)

0.999702

Solution to Exercise 6.6.66 (p. 369)

0.2528

Solution to Exercise 6.6.68 (p. 369)

2.9544

Solution to Exercise 6.6.70 (p. 369)

$16.24

Solution to Exercise 6.6.72 (p. 369)

0.24

Solution to Exercise 6.6.74 (p. 370)

0.006099

Solution to Exercise 6.6.76 (p. 370)

23.295102

Solution to Exercise 6.6.78 (p. 370)

0.000144

Solution to Exercise 6.6.80 (p. 370)

0.0000018

Solution to Exercise 6.6.82 (p. 370)

0.0000000471

Available for free at Connexions <http://cnx.org/content/col10615/1.4>

411

Solution to Exercise 6.6.84 (p. 370)

0Solution to Exercise 6.6.86 (p. 370)

10

77

Solution to Exercise 6.6.88 (p. 370)

1.78

Solution to Exercise 6.7.1 (p. 373)

61.5

Solution to Exercise 6.7.2 (p. 373)

1.884

Solution to Exercise 6.7.3 (p. 373)

0.0741

Solution to Exercise 6.7.4 (p. 373)

0.000062

Solution to Exercise 6.7.5 (p. 375)

2.96

Solution to Exercise 6.7.6 (p. 375)

4.58

Solution to Exercise 6.7.7 (p. 375)

40,000

Solution to Exercise 6.7.8 (p. 375)

816.241

Solution to Exercise 6.7.9 (p. 375)

81.6241

Solution to Exercise 6.7.10 (p. 375)

8.16241

Solution to Exercise 6.7.11 (p. 376)

0.816241

Solution to Exercise 6.7.12 (p. 377)

2.98

Solution to Exercise 6.7.13 (p. 377)

0.005

Solution to Exercise 6.7.14 (p. 377)

3.5197645 is an approximate result. Rounding to four decimal places, we get 3.5198

Solution to Exercise 6.7.15 (p. 379)

18.25

Solution to Exercise 6.7.16 (p. 379)

1.825

Solution to Exercise 6.7.17 (p. 379)

0.1825

Solution to Exercise 6.7.18 (p. 379)

0.01825

Solution to Exercise 6.7.19 (p. 379)

6.4618

Solution to Exercise 6.7.20 (p. 379)

0.021926

Solution to Exercise 6.7.21 (p. 379)

1.6

Solution to Exercise 6.7.23 (p. 379)

3.7

Available for free at Connexions <http://cnx.org/content/col10615/1.4>

412

CHAPTER 6. DECIMALS

Solution to Exercise 6.7.25 (p. 380)

6.04

Solution to Exercise 6.7.27 (p. 380)

9.38

Solution to Exercise 6.7.29 (p. 380)

0.46

Solution to Exercise 6.7.31 (p. 380)

8.6

Solution to Exercise 6.7.33 (p. 380)

2.4

Solution to Exercise 6.7.35 (p. 380)

6.21

Solution to Exercise 6.7.37 (p. 380)

2.18

Solution to Exercise 6.7.39 (p. 380)

1.001

Solution to Exercise 6.7.41 (p. 380)

4Solution to Exercise 6.7.43 (p. 380)

14

Solution to Exercise 6.7.45 (p. 380)

111

Solution to Exercise 6.7.47 (p. 381)

643.51006

Solution to Exercise 6.7.49 (p. 381)

0.064351006

Solution to Exercise 6.7.51 (p. 381)

Actual Quotient Tenths Hundredths Thousandths

1.81

1.8

1.81

1.810

Table 6.38

Solution to Exercise 6.7.53 (p. 381)

Actual Quotient Tenths Hundredths Thousandths

4.821

4.8

4.82

4.821

Table 6.39

Solution to Exercise 6.7.55 (p. 381)

Actual Quotient Tenths Hundredths Thousandths

0.00351

0.0

0.00

0.004

Table 6.40

Available for free at Connexions <http://cnx.org/content/col10615/1.4>

413

Solution to Exercise 6.7.57 (p. 382)

0.01

Solution to Exercise 6.7.59 (p. 382)

12.11 months

Solution to Exercise 6.7.61 (p. 382)

27.2 miles per gallon

Solution to Exercise 6.7.63 (p. 382)

3.7

Solution to Exercise 6.7.65 (p. 382)

1.94

Solution to Exercise 6.7.67 (p. 382)

29.120

Solution to Exercise 6.7.69 (p. 382)

0.173

Solution to Exercise 6.7.71 (p. 382)

1.111

Solution to Exercise 6.7.73 (p. 383)

39

8

Solution to Exercise 6.7.75 (p. 383)

47 or 117

30

30

Solution to Exercise 6.7.77 (p. 383)

2.8634

Solution to Exercise 6.8.1 (p. 386)

0.3

Solution to Exercise 6.8.2 (p. 386)

0.83

Solution to Exercise 6.8.3 (p. 386)

1.2

Solution to Exercise 6.8.4 (p. 386)

1.8

Solution to Exercise 6.8.5 (p. 386)

1.17

Solution to Exercise 6.8.6 (p. 386)

36.3636

Solution to Exercise 6.8.7 (p. 387)

0.4

Solution to Exercise 6.8.9 (p. 387)

0.16

Solution to Exercise 6.8.11 (p. 387)

0.142857

Solution to Exercise 6.8.13 (p. 387)

10.526

Solution to Exercise 6.8.15 (p. 387)

0.112

Solution to Exercise 6.8.17 (p. 387)

6.21951

Solution to Exercise 6.8.19 (p. 387)

0.0081

Solution to Exercise 6.8.21 (p. 387)

0.835

Available for free at Connexions <http://cnx.org/content/col10615/1.4>

414

CHAPTER 6. DECIMALS

Solution to Exercise 6.8.23 (p. 387)

0.3

Solution to Exercise 6.8.25 (p. 387)

2.05

Solution to Exercise 6.8.27 (p. 387)

0.7

Solution to Exercise 6.8.29 (p. 388)

0.518

Solution to Exercise 6.8.31 (p. 388)

0.045

Solution to Exercise 6.8.33 (p. 388)

0.72

Solution to Exercise 6.8.35 (p. 388)

0.7

Solution to Exercise 6.8.37 (p. 388)

1Solution to Exercise 6.8.39 (p. 388)

4410

Solution to Exercise 6.8.41 (p. 388)

8.6

Solution to Exercise 6.9.1 (p. 390)

0.25

Solution to Exercise 6.9.2 (p. 391)

0.04

Solution to Exercise 6.9.3 (p. 391)

0.17

Solution to Exercise 6.9.4 (p. 391)

0.9375

Solution to Exercise 6.9.5 (p. 391)

0.95

Solution to Exercise 6.9.6 (p. 391)

8.0126375

Solution to Exercise 6.9.7 (p. 391)

0.5

Solution to Exercise 6.9.9 (p. 391)

0.875

Solution to Exercise 6.9.11 (p. 391)

0.6

Solution to Exercise 6.9.13 (p. 391)

0.04

Solution to Exercise 6.9.15 (p. 391)

0.05

Solution to Exercise 6.9.17 (p. 391)

0.02

Solution to Exercise 6.9.19 (p. 391)

0.3

Solution to Exercise 6.9.21 (p. 391)

0.1875

Solution to Exercise 6.9.23 (p. 392)

0.037

Available for free at Connexions <http://cnx.org/content/col10615/1.4>

415

Solution to Exercise 6.9.25 (p. 392)

0.538461

Solution to Exercise 6.9.27 (p. 392)

7.6

Solution to Exercise 6.9.29 (p. 392)

1.13

Solution to Exercise 6.9.31 (p. 392)

101.24

Solution to Exercise 6.9.33 (p. 392)

0.24125

Solution to Exercise 6.9.35 (p. 392)

810.31063125

Solution to Exercise 6.9.37 (p. 392)

0.11111

Solution to Exercise 6.9.39 (p. 392)

0.33333

Solution to Exercise 6.9.41 (p. 392)

0.55556

Solution to Exercise 6.9.43 (p. 392)

0.77778

Solution to Exercise 6.9.45 (p. 393)

0.09091

Solution to Exercise 6.9.47 (p. 393)

0.27273

Solution to Exercise 6.9.49 (p. 393)

0.45455

Solution to Exercise 6.9.51 (p. 393)

0.63636

Solution to Exercise 6.9.53 (p. 393)

0.81818

Solution to Exercise 6.9.55 (p. 393)

0.128

Solution to Exercise 6.9.57 (p. 393)

0.9746

Solution to Exercise 6.9.59 (p. 393)

0.0002

Solution to Exercise 6.9.61 (p. 393)

38.7347

Solution to Exercise 6.9.63 (p. 394)

15

16

Solution to Exercise 6.9.65 (p. 394)

3 29

40 or 3.725

Solution to Exercise 6.10.1 (p. 395)

2.2 or 215

Solution to Exercise 6.10.2 (p. 395)

9.23

Solution to Exercise 6.10.3 (p. 395)

10

Solution to Exercise 6.10.4 (p. 395)

1

25 or 0.04

Available for free at Connexions <http://cnx.org/content/col10615/1.4>

416

CHAPTER 6. DECIMALS

Solution to Exercise 6.10.5 (p. 396)

1Solution to Exercise 6.10.7 (p. 396)

0.112

Solution to Exercise 6.10.9 (p. 396)

0.055

Solution to Exercise 6.10.11 (p. 396)

0.1276

Solution to Exercise 6.10.13 (p. 396)

0.7

Solution to Exercise 6.10.15 (p. 396)

9.4625

Solution to Exercise 6.10.17 (p. 396)

1.4075

Solution to Exercise 6.10.19 (p. 396)

0.1875

Solution to Exercise 6.10.21 (p. 396)

0.75

Solution to Exercise 6.10.23 (p. 396)

0.615

Solution to Exercise 6.10.25 (p. 396)

1.35

Solution to Exercise 6.10.27 (p. 397)

0.125

Solution to Exercise 6.10.29 (p. 397)

0.15

Solution to Exercise 6.10.31 (p. 397)

2.6

Solution to Exercise 6.10.33 (p. 397)

yes

Solution to Exercise 6.10.35 (p. 397)

23 · 52 · 7 · 11

Solution to Exercise 6.10.37 (p. 397)

0.592

Solution to Exercise 6.12.1 (p. 399)

hundredths

Solution to Exercise 6.12.3 (p. 399)

seven and two tenths

Solution to Exercise 6.12.5 (p. 399)

sixteen and fty-two hundredths

Solution to Exercise 6.12.7 (p. 399)

ve thousandths

Solution to Exercise 6.12.9 (p. 399)

9.12

Solution to Exercise 6.12.11 (p. 399)

56.0035

Solution to Exercise 6.12.13 (p. 399)

0.004081

Solution to Exercise 6.12.15 (p. 400)

85 63

100

Available for free at Connexions <http://cnx.org/content/col10615/1.4>

417

Solution to Exercise 6.12.17 (p. 400)

11

75

Solution to Exercise 6.12.19 (p. 400)

4 7

675

Solution to Exercise 6.12.21 (p. 400)

4.09

Solution to Exercise 6.12.23 (p. 400)

17

Solution to Exercise 6.12.25 (p. 400)

1Solution to Exercise 6.12.27 (p. 400)

8.946

Solution to Exercise 6.12.29 (p. 400)

7.942

Solution to Exercise 6.12.31 (p. 401)

0.03

Solution to Exercise 6.12.33 (p. 401)

181,600.1

Solution to Exercise 6.12.35 (p. 401)

1.4

Solution to Exercise 6.12.37 (p. 401)

1.5079

Solution to Exercise 6.12.39 (p. 401)

2.372

Solution to Exercise 6.12.41 (p. 401)

0.000

Solution to Exercise 6.12.43 (p. 401)

0.12

Solution to Exercise 6.12.45 (p. 401)

2.702

Solution to Exercise 6.12.47 (p. 401)

0.43

Solution to Exercise 6.12.49 (p. 401)

9.571428

Solution to Exercise 6.12.51 (p. 402)

1.3

Solution to Exercise 6.12.53 (p. 402)

125.125125 (not repeating)

Solution to Exercise 6.12.55 (p. 402)

0.083

Solution to Exercise 6.12.57 (p. 402)

0.255

Solution to Exercise 6.12.59 (p. 402)

5.425

Solution to Exercise 6.12.61 (p. 402)

0.09343

Solution to Exercise 6.13.1 (p. 402)

thousandth

Solution to Exercise 6.13.2 (p. 402)

fteen and thirty-six thousandths

Available for free at Connexions <http://cnx.org/content/col10615/1.4>

418

CHAPTER 6. DECIMALS

Solution to Exercise 6.13.3 (p. 402)

81.12

Solution to Exercise 6.13.4 (p. 402)

0.003017

Solution to Exercise 6.13.5 (p. 402)

39

50

Solution to Exercise 6.13.6 (p. 403)

7

8

Solution to Exercise 6.13.7 (p. 403)

4.8

Solution to Exercise 6.13.8 (p. 403)

200

Solution to Exercise 6.13.9 (p. 403)

0.07

Solution to Exercise 6.13.10 (p. 403)

20.997

Solution to Exercise 6.13.11 (p. 403)

45.565

Solution to Exercise 6.13.12 (p. 403)

77.76

Solution to Exercise 6.13.13 (p. 403)

16

Solution to Exercise 6.13.14 (p. 403)

2.4

Solution to Exercise 6.13.15 (p. 403)

1.1256

Solution to Exercise 6.13.16 (p. 403)

0Solution to Exercise 6.13.17 (p. 403)

6.18

Solution to Exercise 6.13.18 (p. 403)

0.055625

Solution to Exercise 6.13.19 (p. 403)

5.45

Solution to Exercise 6.13.20 (p. 403)

27

128 or 0.2109375

Available for free at Connexions <http://cnx.org/content/col10615/1.4>

Chapter 7

Ratios and Rates

7.1 Objectives1

After completing this chapter, you should

Ratios and Rates (Section 7.2)

• be able to distinguish between denominate and pure numbers and between ratios and rates Proportions (Section 7.3)

• be able to describe proportions and nd the missing factor in a proportion

• be able to work with proportions involving rates

Applications of Proportions (Section 7.4)

• solve proportion problems using the ve-step method

Percent (Section 7.5)

• understand the relationship between ratios and percents

• be able to make conversions between fractions, decimals, and percents Fractions of One Percent (Section 7.6)

• understand the meaning of a fraction of one percent

• be able to make conversions involving fractions of one percent

Applications of Percents (Section 7.7)

• be able to distinguish between base, percent, and percentage

• be able to nd the percentage, the percent, and the base

7.2 Ratios and Rates2

7.2.1 Section Overview

• Denominate Numbers and Pure Numbers

• Ratios and Rates

1This content is available online at <http://cnx.org/content/m18895/1.3/>.

2This content is available online at <http://cnx.org/content/m34980/1.2/>.

Available for free at Connexions <http://cnx.org/content/col10615/1.4> 419

Image 321

420

CHAPTER 7. RATIOS AND RATES

7.2.2 Denominate Numbers and Pure Numbers

Denominate Numbers, Like and Unlike Denominate Numbers

It is often necessary or convenient to compare two quantities. Denominate numbers are numbers together with some specied unit. If the units being compared are alike, the denominate numbers are called like denominate numbers. If units are not alike, the numbers are called unlike denominate numbers.

Examples of denominate numbers are shown in the diagram:

Pure Numbers

Numbers that exist purely as numbers and do not represent amounts of quantities are called pure numbers.

Examples of pure numbers are 8, 254, 0, 215, 2, and 0.07.

8

5

Numbers can be compared in two ways: subtraction and division.

Comparing Numbers by Subtraction and Division

Comparison of two numbers by subtraction indicates how much more one number is than another.

Comparison by division indicates how many times larger or smaller one number is than another.

Comparing Pure or Like Denominate Numbers by Subtraction

Numbers can be compared by subtraction if and only if they both are like denominate numbers or both pure numbers.

7.2.2.1 Sample Set A

Example 7.1

Compare 8 miles and 3 miles by subtraction.

8 mile − 3 miles = 5 miles

This means that 8 miles is 5 miles more than 3 miles.

Examples of use: I can now jog 8 miles whereas I used to jog only 3 miles. So, I can now jog 5

miles more than I used to.

Example 7.2

Compare 12 and 5 by subtraction.

12 − 5 = 7

This means that 12 is 7 more than 5.

Example 7.3

Comparing 8 miles and 5 gallons by subtraction makes no sense.

8 miles − 5 gallons =?

Example 7.4

Compare 36 and 4 by division.

36 ÷ 4 = 9

This means that 36 is 9 times as large as 4. Recall that 36 ÷ 4 = 9 can be expressed as 36 = 9.

4

Available for free at Connexions <http://cnx.org/content/col10615/1.4>

421

Example 7.5

Compare 8 miles and 2 miles by division.

8 miles

2 miles = 4

This means that 8 miles is 4 times as large as 2 miles.

Example of use: I can jog 8 miles to your 2 miles. Or, for every 2 miles that you jog, I jog 8. So, I jog 4 times as many miles as you jog.

Notice that when like quantities are being compared by division, we drop the units. Another way of looking at this is that the units divide out (cancel).

Example 7.6

Compare 30 miles and 2 gallons by division.

30 miles

2 gallons = 15 miles

1 gallon

Example of use: A particular car goes 30 miles on 2 gallons of gasoline. This is the same as getting 15 miles to 1 gallon of gasoline.

Notice that when the quantities being compared by division are unlike quantities, we do not drop the units.

7.2.2.2 Practice Set A

Make the following comparisons and interpret each one.

Exercise 7.2.1

(Solution on p. 466.)

Compare 10 diskettes to 2 diskettes by

(a) subtraction:

(b) division:

Exercise 7.2.2

(Solution on p. 466.)

Compare, if possible, 16 bananas and 2 bags by

(a) subtraction:

(b) division:

7.2.3 Ratios and Rates

Ratio

A comparison, by division, of two pure numbers or two like denominate numbers is a ratio.

The comparison by division of the pure numbers 36 and the like denominate numbers 8 miles 4

2 miles are examples

of ratios.

Rate

A comparison, by division, of two unlike denominate numbers is a rate.

The comparison by division of two unlike denominate numbers, such as

55 miles

1 gallon and 40 dollars

5 tickets

Available for free at Connexions <http://cnx.org/content/col10615/1.4>

422

CHAPTER 7. RATIOS AND RATES

are examples of rates.

Let’s agree to represent two numbers (pure or denominate) with the letters a and b. This means that we’re letting a represent some number and b represent some, perhaps dierent, number. With this agreement, we can write the ratio of the two numbers a and b as

a or b

b

a

The ratio a is read as ” a to b.”

b

The ratio b is read as ” b to a.”

a

Since a ratio or a rate can be expressed as a fraction, it may be reducible.

7.2.3.1 Sample Set B

Example 7.7

The ratio 30 to 2 can be expressed as 30. Reducing, we get 15.

2

1

The ratio 30 to 2 is equivalent to the ratio 15 to 1.

Example 7.8

The rate “4 televisions to 12 people” can be expressed as 4 televisions 12 people . The meaning of this rate is

that “for every 4 televisions, there are 12 people.”

Reducing, we get 1 television

3 people . The meaning of this rate is that “for every 1 television, there are 3

people.

Thus, the rate of “4 televisions to 12 people” is the same as the rate of “1 television to 3 people.”

7.2.3.2 Practice Set B

Write the following ratios and rates as fractions.

Exercise 7.2.3

(Solution on p. 466.)

3 to 2

Exercise 7.2.4

(Solution on p. 466.)

1 to 9

Exercise 7.2.5

(Solution on p. 466.)

5 books to 4 people

Exercise 7.2.6

(Solution on p. 466.)

120 miles to 2 hours

Exercise 7.2.7

(Solution on p. 466.)

8 liters to 3 liters

Write the following ratios and rates in the form “a to b.” Reduce when necessary.

Exercise 7.2.8

(Solution on p. 466.)

9

5

Exercise 7.2.9

(Solution on p. 466.)

1

3

Exercise 7.2.10

(Solution on p. 466.)

25 miles

2 gallons

Available for free at Connexions <http://cnx.org/content/col10615/1.4>

423

Exercise 7.2.11

(Solution on p. 466.)

2 mechanics

4 wrenches

Exercise 7.2.12

(Solution on p. 466.)

15 video tapes

18 video tapes

7.2.4 Exercises

For the following 9 problems, complete the statements.

Exercise 7.2.13

(Solution on p. 466.)

Two numbers can be compared by subtraction if and only if

.

Exercise 7.2.14

A comparison, by division, of two pure numbers or two like denominate numbers is called a

.

Exercise 7.2.15

(Solution on p. 466.)

A comparison, by division, of two unlike denominate numbers is called a

.

Exercise 7.2.16

6

11 is an example of a

. (ratio/rate)

Exercise 7.2.17

(Solution on p. 466.)

5

12 is an example of a

. (ratio/rate)

Exercise 7.2.18

7 erasers

12 pencils is an example of a

. (ratio/rate)

Exercise 7.2.19

(Solution on p. 466.)

20 silver coins

35 gold coins is an example of a

.(ratio/rate)

Exercise 7.2.20

3 sprinklers

5 sprinklers is an example of a

. (ratio/rate)

Exercise 7.2.21

(Solution on p. 466.)

18 exhaust valves

11 exhaust valves is an example of a

.(ratio/rate)

For the following 7 problems, write each ratio or rate as a verbal phrase.

Exercise 7.2.22

8

3

Exercise 7.2.23

(Solution on p. 466.)

2

5

Exercise 7.2.24

8 feet

3 seconds

Exercise 7.2.25

(Solution on p. 466.)

29 miles

2 gallons

Exercise 7.2.26

30,000 stars

300 stars

Exercise 7.2.27

(Solution on p. 466.)

5 yards

2 yards

Exercise 7.2.28

164 trees

28 trees

For the following problems, write the simplied fractional form of each ratio or rate.

Exercise 7.2.29

(Solution on p. 466.)

12 to 5

Available for free at Connexions <http://cnx.org/content/col10615/1.4>

424

CHAPTER 7. RATIOS AND RATES

Exercise 7.2.30

81 to 19

Exercise 7.2.31

(Solution on p. 466.)

42 plants to 5 homes

Exercise 7.2.32

8 books to 7 desks

Exercise 7.2.33

(Solution on p. 466.)

16 pints to 1 quart

Exercise 7.2.34

4 quarts to 1 gallon

Exercise 7.2.35

(Solution on p. 467.)

2.54 cm to 1 in

Exercise 7.2.36

80 tables to 18 tables

Exercise 7.2.37

(Solution on p. 467.)

25 cars to 10 cars

Exercise 7.2.38

37 wins to 16 losses

Exercise 7.2.39

(Solution on p. 467.)

105 hits to 315 at bats

Exercise 7.2.40

510 miles to 22 gallons

Exercise 7.2.41

(Solution on p. 467.)

1,042 characters to 1 page

Exercise 7.2.42

1,245 pages to 2 books

7.2.4.1 Exercises for Review

Exercise 7.2.43

(Solution on p. 467.)

(Section 4.3) Convert 16 to a mixed number.

3

Exercise 7.2.44

(Section 4.7) 15 of 24 is what number?

9

7

Exercise 7.2.45

(Solution on p. 467.)

(Section 5.3) Find the dierence. 11

28 − 7

45 .

Exercise 7.2.46

(Section 6.8) Perform the division. If no repeating patterns seems to exist, round the quotient to three decimal places: 22.35 ÷ 17

Exercise 7.2.47

(Solution on p. 467.)

(Section 6.10) Find the value of 1.85 + 3 · 4.1

8

Available for free at Connexions <http://cnx.org/content/col10615/1.4>

Image 322

425

7.3 Proportions3

7.3.1 Section Overview

• Ratios, Rates, and Proportions

• Finding the Missing Factor in a Proportion

• Proportions Involving Rates

7.3.2 Ratios, Rates, and Proportions

Ratio, Rate

We have dened a ratio as a comparison, by division, of two pure numbers or two like denominate numbers.

We have dened a rate as a comparison, by division, of two unlike denominate numbers.

Proportion

A proportion is a statement that two ratios or rates are equal. The following two examples show how to read proportions.

7.3.2.1 Sample Set A

Write or read each proportion.

Example 7.9

3 = 12

5

20

3 is to 5 as 12 is to 20

Example 7.10

10 items

5 dollars = 2 items

1 dollar

10 items is to 5 dollars as 2 items is to 1 dollar

Example 7.11

8 is to 12 as 16 is to 24.

8

12 = 16

24

Example 7.12

50 milligrams of vitamin C is to 1 tablet as 300 milligrams of vitamin C is to 6 tablets.

50 = 300

1

6

3This content is available online at <http://cnx.org/content/m34981/1.2/>.

Available for free at Connexions <http://cnx.org/content/col10615/1.4>

Image 323

426

CHAPTER 7. RATIOS AND RATES

7.3.2.2 Practice Set A

Write or read each proportion.

Exercise 7.3.1

(Solution on p. 467.)

3 = 6

8

16

Exercise 7.3.2

(Solution on p. 467.)

2 people

1 window = 10 people

5 windows

Exercise 7.3.3

(Solution on p. 467.)

15 is to 4 as 75 is to 20.

Exercise 7.3.4

(Solution on p. 467.)

2 plates are to 1 tray as 20 plates are to 10 trays.

7.3.3 Finding the Missing Factor in a Proportion

Many practical problems can be solved by writing the given information as proportions. Such proportions will be composed of three specied numbers and one unknown number. It is customary to let a letter, such as x, represent the unknown number. An example of such a proportion is

x = 20

4

16

This proportion is read as ” x is to 4 as 20 is to 16.”

There is a method of solving these proportions that is based on the equality of fractions. Recall that two fractions are equivalent if and only if their cross products are equal. For example, Notice that in a proportion that contains three specied numbers and a letter representing an unknown quantity, that regardless of where the letter appears, the following situation always occurs.

(number) · (letter) = (number) · (number)

|

{z

}

We recognize this as a multiplication statement. Specically, it is a missing factor statement. (See Section 4.7

for a discussion of multiplication statements.) For example,

x = 20

4

16

means that 16 · x = 4 · 20

4 = 16

means that 4 · 20 = 16 · x

x

20

5 = x

means that 5 · 16 = 4 · x

4

16

5 = 20

means that 5 · x = 4 · 20

4

x

Each of these statements is a multiplication statement. Specically, each is a missing factor statement. (The letter used here is x, whereas M was used in Section 4.7.)

Finding the Missing Factor in a Proportion

The missing factor in a missing factor statement can be determined by dividing the product by the known factor, that is, if x represents the missing factor, then

x = (product) ÷ (known factor)

Available for free at Connexions <http://cnx.org/content/col10615/1.4>

427

7.3.3.1 Sample Set B

Find the unknown number in each proportion.

Example 7.13

x = 20

4

16 . Find the cross product.

16 · x

=

20 · 4

16 · x

=

80

Divide the product 80 by the known factor 16.

x

=

80

16

x

=

5

The unknown number is 5.

This mean that 5 = 20

4

16 , or 5 is to 4 as 20 is to 16.

Example 7.14

5 = 20

x

16 . Find the cross product.

5 · 16

=

20 · x

80

=

20 · x

Divide the product 80 by the known factor 20.

80

=

x

20

4

=

x

The unknown number is 4.

This means that 5 = 20

4

16 , or, 5 is to 4 as 20 is to 6.

Example 7.15

16 = 64 Find the cross product.

3

x

16 · x

=

64 · 3

16 · x

=

192

Divide 192 by 16.

x

=

192

16

x

=

12

The unknown number is 12.

The means that 16 = 64

3

12 , or, 16 is to 3 as 64 is to 12.

Example 7.16

9 = x

8

40 Find the cross products.

9 · 40

=

8 · x

360

=

8 · x

Divide 360 by 8.

360

=

x

8

45

=

x

The unknown number is 45.

7.3.3.2 Practice Set B

Find the unknown number in each proportion.

Exercise 7.3.5

(Solution on p. 467.)

x = 12

8

32

Available for free at Connexions <http://cnx.org/content/col10615/1.4>

Image 324

Image 325

Image 326

Image 327

428

CHAPTER 7. RATIOS AND RATES

Exercise 7.3.6

(Solution on p. 467.)

7 = 14

x

10

Exercise 7.3.7

(Solution on p. 467.)

9

11 = x

55

Exercise 7.3.8

(Solution on p. 467.)

1 = 8

6

x

7.3.4 Proportions Involving Rates

Recall that a rate is a comparison, by division, of unlike denominate numbers. We must be careful when setting up proportions that involve rates. The form is important. For example, if a rate involves two types of units, say unit type 1 and unit type 2, we can write

or

Both cross products produce a statement of the type

(unit type 1) · (unit type 2) = (unit type 1) · (unit type 2)

which we take to mean the comparison

Examples of correctly expressed proportions are the following:

However, if we write the same type of units on dierent sides, such as,

unit type 1

unit type 2 = unit type 2

unit type 1

the cross product produces a statement of the form

Available for free at Connexions <http://cnx.org/content/col10615/1.4>

Image 328

429

We can see that this is an incorrect comparison by observing the following example: It is incorrect to write 2 hooks

3 poles = 6 poles

4 hooks

for two reason.

1. The cross product is numerically wrong: (2 · 4 6= 3 · 6).

2. The cross product produces the statement hooks are to hooks as poles are to poles, which makes no sense.

7.3.5 Exercises

Exercise 7.3.9

(Solution on p. 467.)

A statement that two ratios or

are equal is called a

.

For the following 9 problems, write each proportion in fractional form.

Exercise 7.3.10

3 is to 7 as 18 is to 42.

Exercise 7.3.11

(Solution on p. 467.)

1 is to 11 as 3 is to 33.

Exercise 7.3.12

9 is to 14 as 27 is to 42.

Exercise 7.3.13

(Solution on p. 467.)

6 is to 90 as 3 is to 45.

Exercise 7.3.14

5 liters is to 1 bottle as 20 liters is to 4 bottles.

Exercise 7.3.15

(Solution on p. 467.)

18 grams of cobalt is to 10 grams of silver as 36 grams of cobalt is to 20 grams of silver.

Exercise 7.3.16

4 cups of water is to 1 cup of sugar as 32 cups of water is to 8 cups of sugar.

Exercise 7.3.17

(Solution on p. 467.)

3 people absent is to 31 people present as 15 people absent is to 155 people present.

Exercise 7.3.18

6 dollars is to 1 hour as 90 dollars is to 15 hours.

For the following 10 problems, write each proportion as a sentence.

Exercise 7.3.19

(Solution on p. 467.)

3 = 15

4

20

Exercise 7.3.20

1 = 5

8

40

Exercise 7.3.21

(Solution on p. 467.)

3 joggers

100 feet = 6 joggers

200 feet

Exercise 7.3.22

12 marshmallows

3 sticks

= 36 marshmallows

9 sticks

Available for free at Connexions <http://cnx.org/content/col10615/1.4>

430

CHAPTER 7. RATIOS AND RATES

Exercise 7.3.23

(Solution on p. 467.)

40 miles

80 miles = 2 gallons

4 gallons

Exercise 7.3.24

4 couches

10 couches = 2 houses

5 houses

Exercise 7.3.25

(Solution on p. 467.)

1 person

1 job

= 8 people

8 jobs

Exercise 7.3.26

1 popsicle

1 popsicle

2

2 children =

1 child

Exercise 7.3.27

(Solution on p. 467.)

2,000 pounds

1 ton

= 60,000 pounds

30 tons

Exercise 7.3.28

1 table

5 tables = 2 people

10 people

For the following 10 problems, solve each proportion.

Exercise 7.3.29

(Solution on p. 468.)

x = 6

5

15

Exercise 7.3.30

x

10 = 28

40

Exercise 7.3.31

(Solution on p. 468.)

5 = 10

x

16

Exercise 7.3.32

13 = 39

x

60

Exercise 7.3.33

(Solution on p. 468.)

1 = x

3

24

Exercise 7.3.34

7

12 = x

60

Exercise 7.3.35

(Solution on p. 468.)

8 = 72

3

x

Exercise 7.3.36

16 = 48

1

x

Exercise 7.3.37

(Solution on p. 468.)

x

25 = 200

125

Exercise 7.3.38

65

30 = x

60

For the following 5 problems, express each sentence as a proportion then solve the proportion.

Exercise 7.3.39

(Solution on p. 468.)

5 hats are to 4 coats as x hats are to 24 coats.

Exercise 7.3.40

x cushions are to 2 sofas as 24 cushions are to 16 sofas.

Exercise 7.3.41

(Solution on p. 468.)

1 spacecraft is to 7 astronauts as 5 spacecraft are to x astronauts.

Exercise 7.3.42

56 microchips are to x circuit boards as 168 microchips are to 3 circuit boards.

Exercise 7.3.43

(Solution on p. 468.)

18 calculators are to 90 calculators as x students are to 150 students.

Exercise 7.3.44

x dollars are to $40,000 as 2 sacks are to 1 sack.

Available for free at Connexions <http://cnx.org/content/col10615/1.4>

431

Indicate whether the proportion is true or false.

Exercise 7.3.45

(Solution on p. 468.)

3

16 = 12

64

Exercise 7.3.46

2

15 = 10

75

Exercise 7.3.47

(Solution on p. 468.)

1 = 3

9

30

Exercise 7.3.48

6 knives

7 forks = 12 knives

15 forks

Exercise 7.3.49

(Solution on p. 468.)

33 miles

1 gallon = 99 miles

3 gallons

Exercise 7.3.50

320 feet

5 seconds = 65 feet

1 second

Exercise 7.3.51

(Solution on p. 468.)

35 students

70 students = 1 class

2 classes

Exercise 7.3.52

9 ml chloride

45 ml chloride = 1 test tube

7 test tubes

7.3.5.1 Exercises for Review

Exercise 7.3.53

(Solution on p. 468.)

(Section 1.7) Use the number 5 and 7 to illustrate the commutative property of addition.

Exercise 7.3.54

(Section 2.7) Use the numbers 5 and 7 to illustrate the commutative property of multiplication.

Exercise 7.3.55

(Solution on p. 468.)

(Section 5.3) Find the dierence. 514 − 322.

Exercise 7.3.56

(Section 6.6) Find the product. 8.06129 · 1, 000.

Exercise 7.3.57

(Solution on p. 468.)

(Section 7.2) Write the simplied fractional form of the rate sixteen sentences to two paragraphs.

7.4 Applications of Proportions4

7.4.1 Section Overview

• The Five-Step Method

• Problem Solving

4This content is available online at <http://cnx.org/content/m34982/1.2/>.

Available for free at Connexions <http://cnx.org/content/col10615/1.4>

432

CHAPTER 7. RATIOS AND RATES

7.4.2 The Five-Step Method

In Section 7.3 we noted that many practical problems can be solved by writing the given information as proportions. Such proportions will be composed of three specied numbers and one unknown number represented by a letter.

The rst and most important part of solving a proportion problem is to determine, by careful reading, what the unknown quantity is and to represent it with some letter.

The Five-Step Method

The ve-step method for solving proportion problems:

1. By careful reading, determine what the unknown quantity is and represent it with some letter. There will be only one unknown in a problem.

2. Identify the three specied numbers.

3. Determine which comparisons are to be made and set up the proportion.

4. Solve the proportion (using the methods of Section 7.3).

5. Interpret and write a conclusion in a sentence with the appropriate units of measure.

Step 1 is extremely important. Many problems go unsolved because time is not taken to establish what quantity is to be found.

When solving an applied problem, always begin by determining the unknown quantity and representing it with a letter.

7.4.3 Problem Solving

7.4.3.1 Sample Set A

Example 7.17

On a map, 2 inches represents 25 miles. How many miles are represented by 8 inches?

Step 1: The unknown quantity is miles.

Let x = number of miles represented by 8 inches

Step 2: The three specied numbers are

2 inches

25 miles

8 inches

Step 3: The comparisons are

2 inches to 25 miles → 2 inches

25 miles

8 inches to x miles → 8 inches

x miles

Proportions involving ratios and rates are more readily solved by suspending the units while doing the computations.

2

25 = 8x

Step 4:

2

=

8

Perform the cross multiplication.

25

x

2 · x

=

8 · 25

2 · x

=

200

Divide 200 by 2.

x

=

200

2

x

=

100

In step 1, we let x represent the number of miles. So, x represents 100 miles.

Step 5: If 2 inches represents 25 miles, then 8 inches represents 100 miles.

Try Exercise 7.4.1 in Section 7.4.3.2 (Practice Set A).

Available for free at Connexions <http://cnx.org/content/col10615/1.4>

433

Example 7.18

An acid solution is composed of 7 parts water to 2 parts acid. How many parts of water are there in a solution composed of 20 parts acid?

Step 1: The unknown quantity is the number of parts of water.

Let n = number of parts of water.

Step 2: The three specied numbers are

7 parts water

2 parts acid

20 parts acid

Step 3: The comparisons are

7 parts water to 2 parts acid → 72

n parts water to 20 parts acid → n

20

7 = n

2

20

Step 4: 7 = n Perform the cross multiplication.

2

20

7 · 20

=

2 · n

140

=

2 · n

Divide 140 by 2.

140

=

n

2

70

=

n

In step 1 we let n represent the number of parts of water. So, n represents 70 parts of water.

Step 5: 7 parts water to 2 parts acid indicates 70 parts water to 20 parts acid.

Try Problem 7.4.2 in Section 7.4.3.2 (Practice Set A).

Example 7.19

A 5-foot girl casts a 31-foot shadow at a particular time of the day. How tall is a person who casts 3

a 3-foot shadow at the same time of the day?

Step 1: The unknown quantity is the height of the person.

Let h = height of the person.

Step 2: The three specied numbers are

5 feet ( height of girl)

3 1 feet (length of shadow)

3

3 feet (length of shadow)

Step 3: The comparisons are

5-foot girl is to 31foot shadow → 5

3

3 1

3

h-foot person is to 3-foot shadow → h3

5

= h

3 1

3

3

Step 4:

5

=

h

3 1

3

3

5 · 3

=

3 1 · h

3

15

=

10 · h Divide 15 by 10

3

3

15

=

h

10

3

3

)15 · 3

=

h

1

)10

2

9

=

h

2

h

=

4 12

Step 5: A person who casts a 3-foot shadow at this particular time of the day is 41 feet tall.

2

Try Exercise 7.4.3 in Section 7.4.3.2 (Practice Set A).

Available for free at Connexions <http://cnx.org/content/col10615/1.4>

434

CHAPTER 7. RATIOS AND RATES

Example 7.20

The ratio of men to women in a particular town is 3 to 5. How many women are there in the town if there are 19,200 men in town?

Step 1: The unknown quantity is the number of women in town.

Let x = number of women in town.

Step 2: The three specied numbers are

3519,200

Step 3: The comparisons are 3 men to 5 women → 35

19,200 men to x women → 19,200

x

3 = 19,200

5

x

Step 4: 3 = 19,200

5

x

3 · x

=

19, 200 · 5

3 · x

=

96, 000

x

=

96,000

3

x

=

32, 000

Step 5: There are 32,000 women in town.

Example 7.21

The rate of wins to losses of a particular baseball team is 9 . How many games did this team lose 2

if they won 63 games?

Step 1: The unknown quantity is the number of games lost.

Let n = number of games lost.

Step 2: Since 9→ means 9 wins to 2 losses, the three specied numbers are 2

9 (wins)

2 (losses)

63 (wins)

Step 3: The comparisons are

9 wins to 2 losses→ 92

63 wins to n losses → 63n

9 = 63

2

n

Step 4: 9 = 63

2

n

9 · n

=

2 · 63

9 · n

=

126

n

=

126

9

n

=

14

Step 5: This team had 14 losses.

Try Exercise 7.4.4 in Section 7.4.3.2 (Practice Set A).

7.4.3.2 Practice Set A

Solve each problem.

Exercise 7.4.1

(Solution on p. 468.)

On a map, 3 inches represents 100 miles. How many miles are represented by 15 inches?

Available for free at Connexions <http://cnx.org/content/col10615/1.4>

435

Step 1:

Step 2:

Step 3:

Step 4:

Step 5:

Exercise 7.4.2

(Solution on p. 468.)

An alcohol solution is composed of 14 parts water to 3 parts alcohol. How many parts of alcohol are in a solution that is composed of 112 parts water?

Step 1:

Step 2:

Step 3:

Step 4:

Step 5:

Exercise 7.4.3

(Solution on p. 468.)

A 51 -foot woman casts a 7-foot shadow at a particular time of the day. How long of a shadow 2

does a 3-foot boy cast at that same time of day?

Step 1:

Step 2:

Step 3:

Step 4:

Step 5:

Exercise 7.4.4

(Solution on p. 468.)

The rate of houseplants to outside plants at a nursery is 4 to 9. If there are 384 houseplants in the nursery, how many outside plants are there?

Step 1:

Step 2:

Step 3:

Step 4:

Step 5:

Exercise 7.4.5

(Solution on p. 468.)

The odds for a particular event occurring are 11 to 2. (For every 11 times the event does occur, it will not occur 2 times.) How many times does the event occur if it does not occur 18 times?

Step 1:

Step 2:

Step 3:

Step 4:

Step 5:

Exercise 7.4.6

(Solution on p. 468.)

The rate of passing grades to failing grades in a particular chemistry class is 7 . If there are 21

2

passing grades, how many failing grades are there?

Step 1:

Step 2:

Step 3:

Step 4:

Step 5:

Available for free at Connexions <http://cnx.org/content/col10615/1.4>

436

CHAPTER 7. RATIOS AND RATES

7.4.4 Exercises

For the following 20 problems, use the ve-step method to solve each problem.

Exercise 7.4.7

(Solution on p. 468.)

On a map, 4 inches represents 50 miles. How many inches represent 300 miles?

Exercise 7.4.8

On a blueprint for a house, 2 inches represents 3 feet. How many inches represent 10 feet?

Exercise 7.4.9

(Solution on p. 468.)

A model is built to 215 scale. If a particular part of the model measures 6 inches, how long is the actual structure?

Exercise 7.4.10

An acid solution is composed of 5 parts acid to 9 parts of water. How many parts of acid are there in a solution that contains 108 parts of water?

Exercise 7.4.11

(Solution on p. 468.)

An alloy contains 3 parts of nickel to 4 parts of silver. How much nickel is in an alloy that contains 44 parts of silver?

Exercise 7.4.12

The ratio of water to salt in a test tube is 5 to 2. How much salt is in a test tube that contains 35

ml of water?

Exercise 7.4.13

(Solution on p. 469.)

The ratio of sulfur to air in a container is 445. How many ml of air are there in a container that contains 207 ml of sulfur?

Exercise 7.4.14

A 6-foot man casts a 4-foot shadow at a particular time of the day. How tall is a person that casts a 3-foot shadow at that same time of the day?

Exercise 7.4.15

(Solution on p. 469.)

A 51-foot woman casts a 11-foot shadow at a particular time of the day. How long a shadow does 2

2

her 31-foot niece cast at the same time of the day?

2

Exercise 7.4.16

A man, who is 6 feet tall, casts a 7-foot shadow at a particular time of the day. How tall is a tree that casts an 84-foot shadow at that same time of the day?

Exercise 7.4.17

(Solution on p. 469.)

The ratio of books to shelves in a bookstore is 350 to 3. How many books are there in a store that has 105 shelves?

Exercise 7.4.18

The ratio of algebra classes to geometry classes at a particular community college is 13 to 2. How many geometry classes does this college oer if it oers 13 algebra classes?

Exercise 7.4.19

(Solution on p. 469.)

The odds for a particular event to occur are 16 to 3. If this event occurs 64 times, how many times would you predict it does not occur?

Exercise 7.4.20

The odds against a particular event occurring are 8 to 3. If this event does occur 64 times, how many times would you predict it does not occur?

Exercise 7.4.21

(Solution on p. 469.)

The owner of a stationery store knows that a 1-inch stack of paper contains 300 sheets. The owner wishes to stack the paper in units of 550 sheets. How many inches tall should each stack be?

Available for free at Connexions <http://cnx.org/content/col10615/1.4>

437

Exercise 7.4.22

A recipe that requires 6 cups of sugar for 15 servings is to be used to make 45 servings. How much sugar will be needed?

Exercise 7.4.23

(Solution on p. 469.)

A pond loses 71 gallons of water every 2 days due to evaporation. How many gallons of water are 2

lost, due to evaporation, in 1 day?

2

Exercise 7.4.24

A photograph that measures 3 inches wide and 41 inches high is to be enlarged so that it is 5

2

inches wide. How high will it be?

Exercise 7.4.25

(Solution on p. 469.)

If 25 pounds of fertilizer covers 400 square feet of grass, how many pounds will it take to cover 500

square feet of grass?

Exercise 7.4.26

Every 11 teaspoons of a particular multiple vitamin, in granular form, contains 0.65 the minimum 2

daily requirement of vitamin C. How many teaspoons of this vitamin are required to supply 1.25

the minimum daily requirement?

7.4.4.1 Exercises for Review

Exercise 7.4.27

(Solution on p. 469.)

(Section 2.2) Find the product, 818 · 0.

Exercise 7.4.28

(Section 4.4) Determine the missing numerator: 815 = N90.

Exercise 7.4.29

(Solution on p. 469.)

(Section 5.6) Find the value of 3 + 4

10

12

19

.

20

Exercise 7.4.30

(Section 6.5) Subtract 0.249 from the sum of 0.344 and 0.612.

Exercise 7.4.31

(Solution on p. 469.)

(Section 7.3) Solve the proportion: 6 = 36

x

30 .

7.5 Percent5

7.5.1 Section Overview

• Ratios and Percents

• The Relationship Between Fractions, Decimals, and Percents Making Conversions 5This content is available online at <http://cnx.org/content/m34983/1.2/>.

Available for free at Connexions <http://cnx.org/content/col10615/1.4>

438

CHAPTER 7. RATIOS AND RATES

7.5.2 Ratios and Percents

Ratio, Percent

We dened a ratio as a comparison, by division, of two pure numbers or two like denominate numbers. A most convenient number to compare numbers to is 100. Ratios in which one number is compared to 100 are called percents. The word percent comes from the Latin word “per centum.” The word “per” means “for each” or “for every,” and the word “centum” means “hundred.” Thus, we have the following denition.

Percent means for each hundred,” or “for every hundred.”

The symbol % is used to represent the word percent.

7.5.2.1 Sample Set A

Example 7.22

The ratio 26 to 100 can be written as 26%. We read 26% as “twenty-six percent.”

Example 7.23

The ratio 165

100 can be written as 165%.

We read 165% as “one hundred sixty-ve percent.”

Example 7.24

The percent 38% can be written as the fraction 38

100 .

Example 7.25

The percent 210% can be written as the fraction 210

100 or the mixed number 2 10

100 or 2.1.

Example 7.26

Since one dollar is 100 cents, 25 cents is 25

100 of a dollar. This implies that 25 cents is 25% of one

dollar.

7.5.2.2 Practice Set A

Exercise 7.5.1

(Solution on p. 469.)

Write the ratio 16 to 100 as a percent.

Exercise 7.5.2

(Solution on p. 469.)

Write the ratio 195 to 100 as a percent.

Exercise 7.5.3

(Solution on p. 469.)

Write the percent 83% as a ratio in fractional form.

Exercise 7.5.4

(Solution on p. 469.)

Write the percent 362% as a ratio in fractional form.

7.5.3 The Relationship Between Fractions, Decimals, and Percents Making Conversions

Since a percent is a ratio, and a ratio can be written as a fraction, and a fraction can be written as a decimal, any of these forms can be converted to any other.

Before we proceed to the problems in Section 7.5.3.1 (Sample Set B) and Section 7.5.3.2 (Practice Set B), let’s summarize the conversion techniques.

Available for free at Connexions <http://cnx.org/content/col10615/1.4>

439

Conversion Techniques Fractions, Decimals, Percents

Available for free at Connexions <http://cnx.org/content/col10615/1.4>

Image 329

Image 330

440

CHAPTER 7. RATIOS AND RATES

To Convert a Fraction

To Convert a Decimal

To Convert a Percent

To a decimal: Divide the numer- To a fraction: Read the decimal To a decimal: Move the decimal ator by the denominator

and reduce the resulting fraction

point 2 places to the left and drop

the % symbol

To a percent: Convert the frac- To a percent: Move the decimal To a fraction: Drop the % sign tion rst to a decimal, then move point 2 places to the right and af- and write the number over 100.

the decimal point 2 places to the x the % symbol

Reduce, if possible.

right and ax the % symbol.

Table 7.1

7.5.3.1 Sample Set B

Example 7.27

Convert 12% to a decimal.

12% = 12

100 = 0.12

Note that

The % symbol is dropped, and the decimal point moves 2 places to the left.

Example 7.28

Convert 0.75 to a percent.

0.75 = 75

100 = 75%

Note that

The % symbol is axed, and the decimal point moves 2 units to the right.

Example 7.29

Convert 3 to a percent.

5

We see in Example 7.28 that we can convert a decimal to a percent. We also know that we can convert a fraction to a decimal. Thus, we can see that if we rst convert the fraction to a decimal, we can then convert the decimal to a percent.

.6

5)3.0

3 →

or 3 = 0.6 = 6

5

5

10 = 60

100 = 60%

3 0

0

Example 7.30

Convert 42% to a fraction.

42% = 42

100 = 21

50

or

Available for free at Connexions <http://cnx.org/content/col10615/1.4>

441

42% = 0.42 = 42

100 = 21

50

7.5.3.2 Practice Set B

Exercise 7.5.5

(Solution on p. 469.)

Convert 21% to a decimal.

Exercise 7.5.6

(Solution on p. 469.)

Convert 461% to a decimal.

Exercise 7.5.7

(Solution on p. 469.)

Convert 0.55 to a percent.

Exercise 7.5.8

(Solution on p. 469.)

Convert 5.64 to a percent.

Exercise 7.5.9

(Solution on p. 469.)

Convert 320 to a percent.

Exercise 7.5.10

(Solution on p. 469.)

Convert 11 to a percent

8

Exercise 7.5.11

(Solution on p. 469.)

Convert 311 to a percent.

7.5.4 Exercises

For the following 12 problems, convert each decimal to a percent.

Exercise 7.5.12

(Solution on p. 469.)

0.25

Exercise 7.5.13

0.36

Exercise 7.5.14

(Solution on p. 469.)

0.48

Exercise 7.5.15

0.343

Exercise 7.5.16

(Solution on p. 469.)

0.771

Exercise 7.5.17

1.42

Exercise 7.5.18

(Solution on p. 469.)

2.58

Exercise 7.5.19

4.976

Exercise 7.5.20

(Solution on p. 470.)

16.1814

Exercise 7.5.21

533.01

Exercise 7.5.22

(Solution on p. 470.)

2

Available for free at Connexions <http://cnx.org/content/col10615/1.4>

442

CHAPTER 7. RATIOS AND RATES

Exercise 7.5.23

14

For the following 10 problems, convert each percent to a decimal.

License

Math Copyright © by sboschman. All Rights Reserved.

Share This Book